Sunteți pe pagina 1din 98

Economics- Subject wise compilation of 60 Day Plan- 2016

Q.1) Which among the following is/are relative merits of GDP?


1. It enables central banks and policymakers to evaluate whether the economy is in recession
or inflation
2. GDP is available for practically every country in the world and allows crude comparisons
between the standard of living in different countries
3. GDP has also held significance as a universal metric, as it includes voluntary market
transactions
Choose the correct code
a) 1 and 2 only
b) 2 only
c) 2 and 3 only
d) 1, 2 and 3
Q.1) Solution (a)
Explanation:
Relative merits of GDP includes:
GDP growth over time enables central banks and policymakers to evaluate whether the
economy is in recession or inflation.
GDP has also held significance as a universal metric over the years, to measure the economic
performance of a nation.
GDP excludes (not includes) voluntary market transactions: GDP fails to account for
productive non-market activities, like a mother taking care of her child, a homemaker doing
household chores, a homeowner doing maintenance of his house, leisure (paid vacation,
holidays, leave time), improvement in product quality, etc.
Q.2) Macroeconomics sees an economy as a combination of 4 sectors. They are
a) Land, labour, capital, entrepreneur
b) Households, firms, government, external sector
c) Consumer, entrepreneur, firms, government
d) Consumer, firms, government, external sector
Q.2) Solution (b)
Explanation:
Macroeconomics deals with the aggregate economic variables of an economy. It also takes
into account various interlinkages which may exist between the different sectors of an
economy. This is what distinguishes it from microeconomics; which mostly examines the
functioning of the particular sectors of the economy, assuming that the rest of the economy
remains the same.
Macroeconomics emerged as a separate subject in the 1930s due to Keynes.
Macroeconomics sees an economy as a combination of four sectors, namely households,
firms, government and external sector.

www.iasbaba.com

Economics- Subject wise compilation of 60 Day Plan- 2016


Q.3) Consider the following statements about Nominal and Real GDP:
1. Nominal GDP refers to the current year production of financial goods and services valued at
base year prices
2. Real GDP refers to the current year production of financial goods and services valued at
current year prices
Which of the above statements is/are not correct?
a) Only 1
b) Only 2
c) Both 1 and 2
d) Neither 1 nor 2
Q.3) Solution (c)
Explanation:
Nominal GDP refers to the current year production of financial goods and services valued at
current year prices
Real GDP refers to the current year production of financial goods and services valued at base
year prices
Q.4) Consider the following statements with regard to GDP and GNP:
1. Gross Domestic Product (GDP), measures the output of goods and services produced within
the geographical boundary of a nation regardless of the residence of that labour or owner of
capital.
2. Gross National Product (GNP) measures the total output of goods and services produced by
residents of a nation regardless of where they live and work or where they own capital.
Which of the above statements is/are correct?
a) Only 1
b) Only 2
c) Both 1 and 2
d) Neither 1 nor 2
Q.4) Solution (c)
Explanation:
Gross Domestic Product
An estimated value of the total worth of a countrys production and services, within its
boundary, by its nationals and foreigners, calculated over the course on one year.
Total value of products & Services produced within the territorial boundary of a country.
GDP is essentially about where production takes place.
Gross National Product
An estimated value of the total worth of production and services, by citizens of a country, on
its land or on foreign land, calculated over the course on one year.
www.iasbaba.com

Economics- Subject wise compilation of 60 Day Plan- 2016

Total value of Goods and Services produced by all nationals of a country (whether within or
outside the country).
GNP is about who produces.

Q.5) Which of the following provides correct definition of GDP deflator?


a) The ratio of GDP at current prices to the GDP at constant prices.
b) GDP at constant prices divided by Wholesale Price Index.
c) The ratio of GDP at constant prices to the GDP at current prices.
d) None of the above.
Q.5) Solution (a)
Explanation:
In economics, the GDP deflator (implicit price deflator) is a measure of the level of prices of
all new, domestically produced, final goods and services in an economy. GDP stands for
gross domestic product, the total value of all final goods and services produced within that
economy during a specified period.
Formula
Where, Nominal GDP is GDP at current prices and Real GDP is GDP at constant prices
(Link: https://upload.wikimedia.org/math/8/1/8/818ffd5a7c33b377f1c98c4f2220c53d.png)
Q.6) Which among the following is correct in regard to Social Progress Index (SPI)?
1. SPI goes beyond the traditional measure of GDP and has most parameters that are required
to fulfill SDGs
2. SPI is based on three fundamental pillars: basic needs for survival; access to the building
blocks to improve living conditions, and access to opportunity to pursue goals and ambitions
3. SPI focuses on outcomes rather than inputs that are used in GDP
4. SPI can best be described as a complementary index to GDP and can be used along with GDP
to achieve social progress
Choose the correct code
a) 1, 2 and 4 only
b) 2 and 4 only
c) 1, 2 and 3 only
d) All the above
Q.6) Solution (d)
Explanation:
About Social Progress Index (SPI)
The Social Progress Index measures the extent to which countries provide for the social and
environmental needs of their citizens.
Fifty-four indicators in the areas of basic human needs, foundations of wellbeing, and
opportunity to progress show the relative performance of nations.

www.iasbaba.com

Economics- Subject wise compilation of 60 Day Plan- 2016

SPI is based on three fundamental pillars: basic needs for survival; access to the building
blocks to improve living conditions, and access to opportunity to pursue goals and
ambitions.
Difference between GDP and SPI:
SPI focuses on outcomes rather than inputs that are used in GDP.
For example, the quality of life and longevity are measured instead of spending on health care,
and peoples experience of discrimination is looked at instead of focusing on whether there is a
law against discrimination.
SPI takes into consideration not just GDP but also inclusive, sustainable growth that will lead
to a significant improvement in peoples lives.
SPI can best be described as a complementary index to GDP and can be used along with GDP
to achieve social progress.
Q.7) Recently union government made some changes to the GDP calculation method. Which
of the following is/are correct with regards to these changes?
1. The base year of estimating national income data is changed from 2004-05 to 2011-12
2. GDP is estimated at market prices, which excludes indirect taxes but includes subsidies
3. The change in base year has pushed up the countrys economic growth as compared to
older series data
Choose the correct code
a) 1 and 2 only
b) 2 only
c) 1 and 3 only
d) 1, 2 and 3
Q.7) Solution (c)
Explanation:
CSO has revised the base year of estimating national income data from 2004-05 to 2011-12
With the revised definition of GDP with new base rate 2011-12, countrys economic growth
got pushed up as compared to older series data
In new definition of the economic growth with new base rate 2011-12, GDP is estimated at
market process, which includes indirect taxes and exclude subsidies
Earlier, GDP growth was estimated at factor cost, which excludes indirect taxes but includes
subsidies
Q.8) The most appropriate measure of a countrys economic growth is its
a) Gross Domestic Product
b) Net Domestic Product
c) Net National Product
d) Per Capita Real Income
Q.8) Solution (d)
www.iasbaba.com

Economics- Subject wise compilation of 60 Day Plan- 2016


Explanation:
Per Capita Real Income is the most appropriate measure of a countrys economic growth
Q.9) Consider the following statements regarding purchasing power parity.
1. It is a technique used to determine the relative value of different currencies.
2. This concept works on the assumption that markets work on the law of one price.
3. This is a popular method used by the IMF and WB in studying the living standards of people
in different economies.
Which of the following is/are correct?
a) 1 and 2 only
b) 1 and 3 only
c) 1 only
d) All of the above
Q.9) Solution (d)
Explanation:
Law of one price, i.e., identical goods and services (in quantity as well as quality) must have
the same price in different markets when measured in a common currency.
Q.10) Which among the following are a part of core industries?
1) Fertilizers
2) Refinery Products
3) Natural Gas
4) Iron
5) Cement
6) Electricity
Choose the correct code:
a) 1,2,4,5 and 6
b) 1,2,3,4 and 6
c) 1,2,3,5 and 6
d) 1,2,3,4,5 and 6
Q.10) Solution (c)
Explanation:
There are eight core sector industries - Coal, Crude Oil, Natural Gas, Refinery Products,
Fertilizers, Steel (Note: No Iron or Iron ore), Cement and Electricity.
Q.11) International Solar Alliance (ISA) will have membership from countries
a) Lying above the Tropic of Cancer
b) Lying between the Tropic of Cancer and the Tropic of Capricorn
c) Lying below the Tropic of Cancer and above the Tropic of Capricorn

www.iasbaba.com

Economics- Subject wise compilation of 60 Day Plan- 2016


d) Along Equator and Tropic of Cancer
Q.11) Solution (b)
India and France have launched an International Solar Alliance to boost solar energy in
developing countries.
The initiative was launched at the UN Climate Change Conference in Paris on 30 November by
Indian Prime Minister Narendera Modi and French President Francois Hollande.
The alliance includes around 121 countries that support the Declaration on the occasion to
launch the international solar alliance of countries dedicated to the promotion of solar energy.
Sourced from- Economic Survey, State of Economy- An Overview
http://www.thehindu.com/sci-tech/energy-and-environment/modi-launches-internationalsolar-alliance/article7934560.ece
Q.12) Sustainable Development Goals (SDGs) and Millennium Development Goals (MDGs)
commonly mentions about
1. Infrastructure
2. Poverty and Hunger
3. Inequality
4. Peace and Justice
5. Water and Sanitation
Select themes common to both, SDGs and MDGs?
a) 1, 2, 3 and 5
b) 2, 3, 4 and 5
c) 2, 3 and 5
d) 1, 2 and 3
Q.12) Solution (a)
Sourced from- Economic Survey, State of Economy- An Overview
Peace and Justice, 16th Goal of SDG- Promote peaceful and inclusive societies for sustainable
development, provide access to justice for all and build effective, accountable and inclusive
institutions at all levels.
MDGs
Goal 1: Eradicate extreme poverty and hunger
Goal 2: Achieve universal primary education
Goal 3: Promote gender equality and empower women
Goal 4: Reduce child mortality rates
Goal 5: Improve maternal health
Goal 6: Combat HIV/AIDS, malaria, and other diseases
Goal 7: Ensure environmental sustainability
Goal 8:Develop a global partnership for development
Q.13) Select the correct statement

www.iasbaba.com

Economics- Subject wise compilation of 60 Day Plan- 2016


a) The Labour Force Participation Rate (LFPR) of women is significantly lower
males in both rural and urban areas.
b) The Labour Force Participation Rate (LFPR) of males is significantly lower
women in both rural and urban areas.
c) The Labour Force Participation Rate (LFPR) of women is significantly lower
males in rural areas but higher in urban areas.
d) The Labour Force Participation Rate (LFPR) of women is significantly lower
males urban areas but higher in rural areas.

than that of
than that of
than that of
than that of

Q.13) Solution (a)


As per the fourth Annual Employment-Unemployment Survey conducted by the Labour Bureau
during the period January 2014 to July 2014, the Labour Force Participation Rate (LFPR) (usual
principal status) is 52.5 for all persons. The LFPR of women is significantly lower than that of
males in both rural and urban areas. The Worker Population Ratio (WPR) reflects similar
patterns.
Sourced from- Economic Survey, State of Economy- An Overview
Q.14) Consider the following statements:
1. CSS funds are released only as central assistance to state plans
2. The plan expenditure of the Centre is greater than the non-plan expenditure.
Choose the correct option from the following
a) Only 1
b) Only 2
c) Both 1 and 2
d) None of the above
Q.14) Solution (d)
CSS funds
Released as central assistance to state plans and also routed through the states budgets.
Provides greater autonomy, authority and responsibility to the states in implementation of
schemes
Recorded a significant increase
Non-plan Expenditure:
Revenue Expenditure accounted for around 92 per cent
Defence capital expenditure the remaining 8 per cent
Constraints in the rationalization of non-plan expenditure: Committed expenditure
1. Interest liability on debt incurred in the past
2. Pension payment to superannuated/retiring workforce from government services

www.iasbaba.com

Economics- Subject wise compilation of 60 Day Plan- 2016

Observe the pie-chart Fig 2.10


Inference: Transport > Energy > Miscc. > Social Services >Industry & minerals
Q.15) Consider the following:
1. Interest and dividend receipts
2. External grants and receipts from fiscal services
3. External grants and receipts from the Union Public Service
4. Disinvestment receipts
Choose the correct option that together constitutes Non-tax revenue in the economy
a) Only 1 and 2
b) 1, 2, and 3
c) 1, 2 , 3 and 4
d) All of the above
Q.15) Solution (b)
Non-debt capital receipts: Recovery of loans and advances, and disinvestment receipts
Q.16) Puga valley in J&k is famous for availability of which of the following energy reserve
a) Coal
b) Geo Thermal
c) Hydro
d) Wind
Q.16) Solution (b)

www.iasbaba.com

Economics- Subject wise compilation of 60 Day Plan- 2016

Q.17) Consider the following statements regarding Remittance


1. India is worlds largest remittance recipient in 2015
2. Indian remittance has declined as compared to last year.
3. While global remittance has decreased, remittance for developing countries increased in
2015
Select the correct code
a) Only 1
b) 1 and 3
c) 1 and 2
d) 1, 2 and 3
Q.17) Solution (d)
As per recently report of World Bank, India remained the worlds largest remittance recipient in
2015. It was revealed by the World Banks annual report Migration and Development Brief. In
2015, India attracted about 69 billion US dollars in remittances, down from 70 billion in 2014

www.iasbaba.com

Economics- Subject wise compilation of 60 Day Plan- 2016


Key Highlights of Report Other large remittance recipients in 2015 were China (64 billion
dollars), Philippines (28 billion), Mexico (25 billion) and Nigeria (21 billion dollar). Global
scenario:
In 2015, global remittances which include those to high-income countries contracted by 1.7% to
581 billion US dollar compared to 592 billion in 2014.
Indian scenario: Remittances to India in 2015 decreased by 2.1 per cent to USD 68.9 billion. This
marks the first decline in remittances since 2009.
Developing countries: Officially recorded remittances to developing countries amounted to
431.6 billion dollars in 2015, an increase of 0.4 per cent over 430 billion dollars in 2014.
Q.18) Which of the following is correct about Mathew effect?
a) The Matthew effect (or accumulated advantage) is the phenomenon where "the rich get
richer and the poor get poorer
b) The Mathew effect means the idea of inflation increasing with employment rates soaring
c) Mathew was a ancient historian who felt rich will help poor and vice versa hence an
equalizing effect
d) Mathew effect is a concept of psychology of people wishing for equality in extreme poverty
Q.18) Solution (a)
The Principle of Cumulative Advantage states that once a social agent gains a small advantage
over other agents, that advantage will compound over time into an increasingly larger
advantage. The effect is well known and is embodied in "the rich get richer and the poor get
poorer". The principle is also known as the Matthew effect.
The term was coined by sociologist Robert Merton in a 1968 paper which described how the
more eminent scientists in a group tend get the most credit for the group's work, regardless of
who did the work.
The Principle of Cumulative Advantage lies at the heart of many large social problems. It's the
causal mechanism for income inequality growth, corruption growth, centralization of power
growth, and hardening of class stratification.

Q.19) Which of the following institutions have their objectives stated to fight poverty in the
world?
1. International Monetary Fund
2. World Bank
3. World Trade Organization
4. World Health Organization
Select the correct answer:
a) 1 only
b) 1 and 2 only
c) 1, 2 and 3 only
d) 1, 2 and 4 only

www.iasbaba.com

10

Economics- Subject wise compilation of 60 Day Plan- 2016

Q.19) Solution (b)


Further reading: https://www.wto.org/english/thewto_e/whatis_e/what_we_do_e.htm
http://www.who.int/about/mission/en/
Q.20) Which of the following are correct about Poverty?
1. Absolute poverty was defined as: a condition characterised by severe deprivation of basic
human needs, including food, safe drinking water, sanitation facilities, health, shelter,
education and information.
2. Relative poverty refers to a standard which is defined in terms of the society in which an
individual lives and which therefore differs between countries and over time.
3. Poverty gap is the mean shortfall of the total population from the poverty line (counting the
nonpoor as having zero shortfall), expressed as a percentage of the poverty line. This
measure reflects the depth of poverty as well as its incidence.
4. Poverty line is the level of income below which one cannot afford to purchase all the
resources one requires to live.
Select the correct answer:
a) 1 and 2 only
b) 1, 2 and 3 only
c) 1, 2 and 4 only
d) All the above
Q.20) Solution (d)
Poverty is a condition where people's basic needs for food, clothing, and shelter are not being
met. Poverty is generally of two types:
(1) Absolute poverty is synonymous with destitution and occurs when people cannot obtain
adequate resources (measured in terms of calories or nutrition) to support a minimum level
of physical health. Absolute poverty means about the same everywhere, and can be
eradicated as demonstrated by some countries.
(2) Relative poverty occurs when people do not enjoy a certain minimum level of living
standards as determined by a government (and enjoyed by the bulk of the population) that
vary from country to country, sometimes within the same country.
Q.21) Arrange following committees w.r.t. Poverty Alleviation in chronological order
1. Lakadwala Committee
2. Y K Alag Committee
3. C Rangarajan Committee
4. Suresh Tendulkar Committee
5. N C Saxena Committee
Select the correct answer:
a) 1 2 3 4 5

www.iasbaba.com

11

Economics- Subject wise compilation of 60 Day Plan- 2016


b) 2 1 4 5 3
c) 1 3 4 5 2
d) 2 5 4 3 1
Q.21) Solution (b)
Further reading: http://planningcommission.nic.in/reports/genrep/pov_rep0707.pdf
Q.22) Consider the following
1. Dr. N.C. Saxena Committee was set up by the Ministry of Rural Development to advise it on
the suitable methodology for BPL Census and not for estimation of poverty.
2. The Planning Commission constituted an Expert Group under the Chairmanship of Professor
S.R. Hashim, with the objective of putting in place a uniform criterion to identify the BPL
households in urban areas so that objectivity and transparency is ensured in delivery of
benefits to the target groups.
Select the correct answer:
a) 1 only
b) 2 only
c) Both 1 and 2
d) None of the above
Q.22) Solution (c)
The Planning Commission constituted an Expert Group under the Chairmanship of Professor
S.R.Hashim to recommend the detailed methodology for identification of families living Below
Poverty Line in urban areas. The Hashim Committee submitted its final report on 24th
December, 2012 to the Planning Commission. In its report, the Hashim Committee
recommended three stage identification process to identify the families living Below Poverty
Line in urban areas which include automatic exclusion, automatic inclusion and scoring index of
the remaining urban families in this order.
Dr. N.C. Saxena Committee was set up by the Ministry of Rural Development to advise it on the
suitable methodology for BPL Census and not for estimation of poverty. However, in the Report
submitted by the Expert Group on 21st August 2009 it is mentioned that the percentage of
people entitled to BPL status should be revised upwards to at least 50%. The committee has
suggested proportionate increase in the state level poverty estimates also. The
recommendations of the Expert Group and other alternative methodologies are being tested
through a pilot socio-economic survey and a Participatory Rural Appraisal (PRA) exercise in
order to finalize the methodology for the final BPL Census.
Further reading: http://pib.nic.in/newsite/PrintRelease.aspx?relid=68126
Q.23) Which of the following are not correctly matched?
1. Lorenz Curve Poverty estimation

www.iasbaba.com

12

Economics- Subject wise compilation of 60 Day Plan- 2016


2. J Curve Taxation
3. Laffer Curve Devalutaion
4. Philips Curve Inflation and uemployment
Select the correct answer:
a) 1 only
b) 1 and 2 only
c) 1, 2 and 3 only
d) All of the above
Q.23) Solution (c)
In economics, the Lorenz curve is a graphical representation of the distribution of income or of
wealth. It was developed by Max O. Lorenz in 1905 for representing inequality of the wealth
distribution.

The J-curve effect is seen in economics when a country's trade balance initially worsens
following a devaluation or depreciation of its currency.
The Phillips curve represents the relationship between the rate of inflation and the
unemployment rate.
In economics, the Laffer curve is a representation of the relationship between rates of
taxation and the resulting levels of government revenue.

Q.24) The Gini coefficient (also known as the Gini index or Giniratio) is a measure of statistical
dispersion intended to represent the income distribution of a nation's residents, and is the
most commonly used measure of inequality. Consider the following about Gini Coefficent.
1. A Gini coefficient of zero expresses perfect equality, where all values are the same (for
example, where everyone has the same income).
2. A Gini coefficient of 1 (or 100%) expresses maximal inequality among values (e.g., for a large
number of people, where only one person has all the income or consumption, and all others
have none, the Gini coefficient will be very nearly one).
Select the correct answer:
a) 1 only
b) 2 only
c) Both 1 and 2
d) None of the above
Q.24) Solution (c)
Further reading: http://data.worldbank.org/indicator/SI.POV.GINI
Q.25) Which of the following is incorrect about the Digital Gender Atlas for Advancing Girls
Education in India?

www.iasbaba.com

13

Economics- Subject wise compilation of 60 Day Plan- 2016


a) In order to plan and execute educational interventions, the purpose of the Gender Atlas is to
help identify and ensure equitable education with a focus on vulnerable girls, including girls
with disabilities.
b) The Ministry of Women and Child Development presented the Digital Gender Atlas for
Advancing Girls Education in India
c) The Atlas provides comparative analysis of individual gender related indicators over three
years and that enables a visual assessment of the change and an understanding of whether
some intervention introduced in a geography at a particular point in time has worked or not.
d) The main components of the Gender Atlas are : (i) Composite Gender Ranking (ii) Trend
Analysis of Gender Indicators (iii) Vulnerabilities based on educational indicators in districts
with substantial tribal, schedule caste, minority population
Q.25) Solution (b)
The Ministry of Human Resource Development, Department of School Education and Literacy,
presented the Digital Gender Atlas for Advancing Girls Education in India. The tool, which has
been developed with the support of UNICEF, will help identify low performing geographic
pockets for girls, particularly from marginalised groups such as scheduled castes, schedule tribes
and Muslim minorities, on specific gender related education indicators.
The Atlas provides comparative analysis of individual gender related indicators over three years
and that enables a visual assessment of the change and an understanding of whether some
intervention introduced in a geography at a particular point in time has worked or not. It is
constructed on an open source platform with an inbuilt scope of updating data by authorized
persons to retain its dynamic character.
Further reading: http://pib.nic.in/newsite/PrintRelease.aspx?relid=116595

Q.26) Consider the following about Employment rates and surveys.


1. Labour Bureau has been entrusted the task of conducting Annual EmploymentUnemployment Survey in the country.
2. The labor force participation rate (LFPR) is the proportion of people eligible to participate in
the labor force who are actually participating in it by working or looking for work. It is
usually expressed as a percent of the total labor force-eligible population in an economy.
3. Unemployment rate is defined most basically as the percentage of the total labor force that
is unemployed but actively seeking employment and willing to work.
Select the correct answer:
a) 1 and 3 only
b) 1 and 2 only
c) 2 and 3 only
d) All of the above

www.iasbaba.com

14

Economics- Subject wise compilation of 60 Day Plan- 2016


Q.26) Solution (d)
Further reading: http://labourbureau.nic.in/Report%20%20Vol%201%20final.pdf
Q.27) Consider the following about Pradhan Mantri Kaushal Vikas Yojana (PMKVY).
1. Pradhan Mantri Kaushal Vikas Yojana (PMKVY) isthe flagship outcome-based skill training
scheme of the new Ministry of Skill Development & Entrepreneurship (MSDE).
2. The objective of this skill certification and reward scheme is to enable and mobilize a large
number of Indian youth to take up outcome based skill training and become employable and
earn their livelihood
3. Under PMKVY, trainees with prior experience or skills and competencies will not be
recognized and they will be given certification only post undergoing assessments.
4. The scheme will be implemented by the Labour Bureau of India.
Select the correct answer about PMKVY:
a) 1 and 2 only
b) 1 and 4 only
c) 2 and 3 only
d) None of the above
Q.27) Solution (a)
Pradhan Mantri Kaushal Vikas Yojanais a unique initiative by the Government of India that aims
to offer 24 lakh Indian youth meaningful, industry relevant, skill based training. Under this
scheme, the trainees will be offered a financial reward and a government certification on
successful completion of training and assessment, which will help them in securing a job for a
better future.
The scheme will be implemented through the National Skill Development Corporation (NSDC).
Further reading:
http://www.skilldevelopment.gov.in/pmkvy.html
http://pmkvyofficial.org/Index.aspx
Q.28) Recently Ministry of Urban development released the second list of would be smart
cities. Consider the following statements with respect to development of green field smart
city.
1. The minimum area for the development of Greenfield smart city is 250 acres
2. GIFT in Gujarat is the best example for Green field development of a smart city
Select the correct option with respect to development of Green field smart city
a) 1 only
b) 2 only
c) Both
d) None

www.iasbaba.com

15

Economics- Subject wise compilation of 60 Day Plan- 2016

Q.28) Solution (c)


Greenfield development will introduce most of the Smart Solutions in a previously vacant
area (more than 250 acres) using innovative planning, plan financing and plan
implementation tools (e.g. land pooling/ land reconstitution) with provision for affordable
housing, especially for the poor. Greenfield developments are required around cities in
order to address the needs of the expanding population. One well known example is the
GIFT City in Gujarat. Unlike retrofitting and redevelopment, greenfield developments could
be located either within the limits of the ULB or within the limits of the local Urban
Development Authority (UDA).
Q.29) Consider the following countries
1. Italy
2. China
3. Russia
4. France
5. Japan
Which of the following countries does not belong to G7 countries?
a) 1, 2 and 3 only
b) 1, 2 and 4 only
c) 2 and 3 only
d) All are members of G7 forum
Q.29) Solution (c)
China and Russia are not members of G7
Q.30) Union cabinet approved National capital goods policy recently. Consider the following
statements with respect to National Capital goods policy
1. The policy aims to create 21 million new jobs by 2025
2. The policy aims to increase exports from the current 27 percent to 40 percent of production.
3. Ministry of commerce and Industry is the nodal agency for the formulation of NCGP 201605-26
Select the correct option
a) 1 and 2 only
b) 2 and 3 only
c) 1 and 3 only
d) All
Q.30) Solution (a)
It is spearheaded by ministry of heavy industries and public enterprises.
http://dhi.nic.in/writereaddata/Content/NationalCapitalGoodsPolicy2016.pdf

www.iasbaba.com

16

Economics- Subject wise compilation of 60 Day Plan- 2016

Q.31) Consider the following statements with respect to PAHAL scheme


1. The scheme was first launched in 2015
2. The scheme encourages the LPG customers to voluntarily give up subsidies
3. The scheme was launched under the aegis of Ministry of Petroleum and natural resources
Select the correct answer
a) 2 and 3 only
b) 3 only
c) 1 and 3 only
d) 1, 2 and 3
Q.31) Solution (b)
The PAHAL (DBTL) scheme was earlier launched on 1st June 2013 and finally covered 291
districts. It required the consumer to mandatorily have an Aadhaar number for availing LPG
Subsidy. The government has comprehensively reviewed the scheme and after examining
the difficulties faced by the consumer substantively modified the scheme prior to launch.
The modified scheme has been re-launched in 54 districts on 15.11.2014 in the 1st Phase
and to be launched in rest of the country on 1.1.2015
Q.32) Recently Global statics about production of e-waste was released. Consider the
following statements with respect to e-waste
1. PCB (polychlorinated biphenyls), Beryllium, Barium are some of the commonly found Ewaste in India
2. India is the largest producer of E waste in the world
Select the correct option
a) 1 only
b) 2 only
c) Both
d) None
Q.32) Solution (a)
U.S., China, Japan and Germany, and India are top e-waste producing countries in the world
Q.33) Consider the following statements with respect to Roads in India
1. Bangalore is one such place in India where the National highway, North south corridor and
Golden Quadrilateral intersect
2. The intersection between North-South and East-West corridor is at Jhansi
Select the correct answer using the code given below
a) 1 only
b) 2 only

www.iasbaba.com

17

Economics- Subject wise compilation of 60 Day Plan- 2016


c) Both 1 and 2
d) Neither of them
Q.33) Solution (c)
Both the statements are true
Q.34) Consider the following statements
1. The trends in enrolment reflect a decline in the percentage of enrolment in government
schools in rural areas
2. In the case of Scheduled Tribe (ST) students, parity between girls and boys has been
achieved across all levels of school and higher education.
Select the correct code
a) Only 1
b) Only 2
c) Both
d) None
Q.34) Solution (a)
In the case of Scheduled Tribe (ST) students, parity between girls and boys has not been
achieved across all levels of school and higher education.
Economic Survey, Social Infrastructure, Employment and Human Development
Q.35) Consider the following regarding Digital Gender Atlas for Advancing Girls Education in
India. It is a tool
1. To help identify low-performing geographic pockets for girls, particularly from marginalized
groups to provides comparative analysis of individual gender-related indicators over the
years.
2. The tool has been developed in partnership with the United Nations Childrens Fund
(UNICEF)
Select the correct code
a) Only 1
b) Only 2
c) Both
d) None
Q.35) Solution (c)
Economic Survey, Social Infrastructure, Employment and Human Development
Q.36) Schemes that are aimed both at creating a conducive environment for industrial
development and doing business with ease and also expanding government support to impart
skill training for workers?

www.iasbaba.com

18

Economics- Subject wise compilation of 60 Day Plan- 2016


1.
2.
3.
4.
5.

Pandit Deendayal Upadhyay Shramev Jayate Karyakram


Shram Suvidha Portal
Random Inspection Scheme
Universal Account Number Scheme
Apprentice Protsahan Yojana

Select the correct code


a) 1, 2 and 3
b) 1, 2, 3, 4, 5
c) 1, 2, 4, 5 and 6
d) 1, 3, 4, 5 and 6
Q.36) Solution (b)
India Year Book- Labor, Skill Development and Employment
Q.37) Consider the following
1. Malaria
2. Dengue
3. Chagas disease
4. Yellow fever
5. Japanese encephalitis
6. Zika
Which of the above are Vector Borne Diseases?
a) 1, 2 and 4
b) 1, 3, 4 and 5
c) 1, 2, 3, 4 and 6
d) 1, 2, 3, 4, 5 and 6
Q.37) Solution (d)
Economic Survey, Social Infrastructure, Employment and Human Development
http://www.who.int/mediacentre/factsheets/fs387/en/
Q.38) The Universal Health Coverage (UHC) index has been developed by
a) WHO
b) World Bank
c) United Nation
d) UNICEF
Q.38) Solution (b)
Economic Survey, Social Infrastructure, Employment and Human Development

www.iasbaba.com

19

Economics- Subject wise compilation of 60 Day Plan- 2016


The Universal Health Coverage (UHC) index has been developed by the World Bank to measure
the progress made in health sectors in select countries of the World. India ranks 143 among 190
countries in terms of per capita expenditure on health ($146 PPP in 2011). It has 157th position
according to per capita government spending on health which is just about $44 PPP. Indias
performance on the indicator on treatment of diarrhoea needs improvement in terms of
enhancing the coverage. The impoverishment indicator reflects the financial risk protection
coverage, with a higher percentage reflecting better coverage.
Q.39) RBI merged two funds to create a new Financial Inclusion Fund (FIF) to support
developmental and promotional activities for expanding reach of banking services. Consider
the following w.r.t FIF
1. The new FIF will be administered by NABARD.
2. FIF is created by merging National Innovation Fund and Financial Inclusion Technology Fund
Select the correct statement/s
a) Only 1
b) Only 2
c) Both
d) None
Q.39) Solution (d)
https://rbi.org.in/Scripts/NotificationUser.aspx?Id=10074&Mode=0
Q.40) Consider the following regarding River Information Services (RIS)
1. They are hardware and software based system, combination of modern tracking equipment
designed to optimize traffic and transport processes in inland navigation.
2. They would facilitate environment protection and better safety provisions for ship to ship
collisions.
Select the correct code
a) Only 1
b) Only 2
c) Both
d) None
Q.40) Solution (b)
http://pib.nic.in/newsite/PrintRelease.aspx?relid=134228
Q.41) Consider the following statements about inflation.
1. Inflation means a persistent rise the price of goods and services.
2. Inflation increases the purchasing power of money.
3. Inflation hurts those with fixed and rising incomes equally.

www.iasbaba.com

20

Economics- Subject wise compilation of 60 Day Plan- 2016


Which of the above are correct?
a) 1 only
b) 2 only
c) 2 and 3 only
d) All of the above
Q.41) Solution (a)
Inflation is the long term rise in the prices of goods and services caused by the devaluation of
currency. Inflationary problems arise when we experience unexpected inflation which is not
adequately matched by a rise in peoples incomes.
If incomes do not increase along with the prices of goods, everyones purchasing power has
been effectively reduced, which can in turn lead to a slowing or stagnant economy. Moreover,
excessive inflation can also wreak havoc on retirement savings as it reduces the purchasing
power of the money that savers and investors have squirreled away.
Q.42) Which of the following are probable effects of inflation?
1. It reduces savings
2. Leads to depreciation of currency
3. Make imports cheap
4. Increases borrowing costs for businesses
Select the correct answer:
a) 1 only
b) 1 and 2 only
c) 1, 2 and 3 only
d) 1, 2 and 4 only
Q.42) Solution (d)
The major effects of inflation are as follows: 1. Effects on Redistribution of Income and Wealth
2. Effects on Production 3. Other Effects!
Inflation affects different people differently. This is because of the fall in the value of money.
When price rises or the value of money falls, some groups of the society gain, some lose and
some stand in-between. Broadly speaking, there are two economic groups in every society, the
fixed income group and the flexible income group.
People belonging to the first group lose and those belonging to the second group gain. The
reason is that the price movements in the case of different goods, services, assets, etc. are not
uniform. When there is inflation, most prices are rising, but the rates of increase of individual
prices differ much. Prices of some goods and services rise faster, of others slowly and of still
others remain unchanged.

www.iasbaba.com

21

Economics- Subject wise compilation of 60 Day Plan- 2016


Q.43) The GDP price deflator is an economic metric that accounts for inflation by converting
output measured at current prices into constant-dollar GDP. Consider the following.
1. It is the ratio of the value of goods and services an economy produces in a particular year at
current prices to that at prices prevailing during any other reference (base) year.
2. The deflator covers the entire range of goods and services produced in the economy as
against the limited commodity baskets for the wholesale or consumer price indices it is
seen as a more comprehensive measure of inflation.
Select the correct answer:
a) 1 only
b) 2 only
c) Both 1 and 2
d) None of the above
Q.43) Solution (c)
The GDP deflator is a measure of price inflation. It is calculated by dividing Nominal GDP by Real
GDP and then multiplying by 100. (Based on the formula). Nominal GDP is the market value of
goods and services produced in an economy, unadjusted for inflation. Real GDP is nominal GDP,
adjusted for inflation to reflect changes in real output. Trends in the GDP deflator are similar to
changes in the Consumer Price Index, which is a different way of measuring inflation.
Further reading: http://indianexpress.com/article/explained/why-deflator-is-best-indicator-ofprice-rise/
Q.44) Which of the following are correct about Producer Price Index?
a) The Producer Price Index (PPI) program measures the change in prices received by the seller.
b) The PPI provides a broader coverage in terms of products and industries and it also includes
services, something which is not a part of the WPI.
c) PPI measures the price pressure due to increase in cost of finished goods.
d) All of the above.
Q.44) Solution (b)
The Producer Price index (PPI) is a family of indexes that measures the average change in selling
prices received by domestic producers of goods and services over time. PPIs measure price
change from the perspective of the seller and differ from other indexes, such as the Consumer
Price Index, that measure price change from the purchaser's perspective. The PPI looks at three
areas of production: industry-based, commodity-based and commodity-based final demand
intermediate demand.
Further reading: http://indianexpress.com/article/business/economy/new-ppi-index-likely-toreplace-wpi-by-fy16/
Q.45) Consider the following.

www.iasbaba.com

22

Economics- Subject wise compilation of 60 Day Plan- 2016


1. Demand Pull inflation involves inflation decreasing as real GDP rises and unemployment
decreases.
2. It can be a result of easy money policy and also referred to as growth inflation.
Select the correct answer:
a) 1 only
b) 2 only
c) Both 1 and 2
d) None of the above
Q.45) Solution (b)
Inflation means a sustained increase in the general price level. However, this increase in the
cost of living can be caused by different factors. The main two types of inflation are
Demand pull inflation this occurs when the economy grows quickly and starts to overheat
Aggregate demand (AD) will be increasing faster than aggregate supply (LRAS).
Cost push inflation this occurs when there is a rise in the price of raw materials, higher
taxes, e.t.c
Demand Pull Inflation - This occurs when AD increases at a faster rate than AS. Demand pull
inflation will typically occur when the economy is growing faster than the long run trend
rate of growth. If demand exceeds supply, firms will respond by pushing up prices.
Q.46) Which of the following is incorrect?
a) Supply shock inflation is caused by reduced supplies due to increased prices of inputs.
b) A type of persistent inflation that occurs by deficiencies in the condition of the economy is
structural inflation.
c) Speculation and cartelization can cause inflation and hence vitiate the market dynamics.
d) None of the above.
Q.46) Solution (d)
This occurs when there is an increase in the cost of production for firms causing aggregate
supply to shift to the left. Cost push inflation could be caused by rising energy and
commodity prices.
Speculation involves trading a financial instrument involving high risk, in expectation of
significant returns. Speculation involves trading a financial instrument involving high risk, in
expectation of significant returns. The motive is to take maximum advantage from
fluctuations in the market.
A cartel is a grouping of producers that work together to protect their interests. Cartels are
created when a few large producers decide to co-operate with respect to aspects of their
market. Once formed, cartels can fix prices for members, so that competition on price is
avoided. In this case cartels are also called price rings. They can also restrict output released
onto the market, such as with OPEC and oil production quotas, and set rules governing other
aspects of the behaviour of members. Setting rules is especially important in oligopolistic

www.iasbaba.com

23

Economics- Subject wise compilation of 60 Day Plan- 2016


markets, as predicted in game theory. A significant attraction of cartels to producers is that
they set rules that members follow, thus reducing risks that would exist without the cartel.
Q.47) Consider the relation of inflation with BOP.
1. Inflation discourages exports as domestic sales are attractive
2. Inflation may erode external competitiveness of domestic products if it leads to higher
production costs.
3. Inflation leads to currency appreciation and hence increases imports.
4. High inflation thus boosts trade and hence solve BOP crisis as such.
Select the correct answer:
a) 1 only
b) 1 and 2 only
c) 1 and 3 only
d) 3 and 4 only
Q.47) Solution (b)
Inflation not only creates problems within the economy, but also in the sphere of external trade
of a country, that is, countrys trade balances with the rest of the World. Countrys trade
relations with the other countries involve exports and imports of goods and services and how
much a country will export and import depends, amongst other thing, on the domestic price
level and variation in it, that is, the rate of inflation.
Countrys transactions with the other countries, which are recorded in balance of payments
(BOPs), get adversely affected if the domestic price rise is high. High rate of inflation in the
domestic market makes domestic goods unattractive to the foreigners and therefore, reduces
demand for exports. Moreover, because of high domestic prices, residents prefer to buy foreign
goods which implies increase in imports. The result of falling exports and increasing imports, on
account of high domestic inflation, is the adverse disequilibrium in the BOPs which, if not kept
within limits, can assume serious proportion and spell a BOPs crisis.
The BOPs crisis, which India experienced in 1991, was of a similar nature. Policy mistakes in the
form of high and unsustainable fiscal deficit financed through creation of new money led to
unprecedented growth in money supply. The resulting inflation entailed high growth in imports
than exports and finally, led to a very serious BOPs crisis involving steep decline in foreign
exchange reserves and possibility of default on external payment front.
Q.48) Which are possible consequences of inflation?
1. Repo rates may go up to squeeze out money.
2. More OMOs may be conducted by RBI by selling G-Secs.
3. Governments fiscal deficit may go up in need to subsidise and make services affordable.
Select the correct answer:

www.iasbaba.com

24

Economics- Subject wise compilation of 60 Day Plan- 2016


a)
b)
c)
d)

2 only
1 and 2 only
2 and 3 only
1 and 3 only

Q.48) Solution (d)


The steps generally taken by the RBI to tackle inflation include a rise in repo rates (the rates at
which banks borrow from the RBI), a rise in Cash Reserve Ratio and a reduction in rate of
interest on cash deposited by banks with RBI. The signals are intended to spur banks to raise
lending rates and to reduce the amount of credit disbursed. The RBI's measures are expected to
suck out a substantial sum from the banks. In effect, while the economy is booming and the
credit needs grow, the central bank is tightening the availability of credit. The RBI also buys
dollars from banks and exporters, partly to prevent the dollars from flooding the market and
depressing the dollar indirectly raising the rupee. In other words, the central bank's
interactions have a desirable objective to keep the rupee devalued which will make India's
exports more competitive, but they increase liquidity.
Q.49) Which of the following is correct about Inflation tax?
a) Inflation tax is a term which refers to the financial loss of value suffered by holders of cash
and as well those on fixed income, due to the effects of inflation;
b) Inflation tax is the tax people pay in developed countries to help governments fight inflation.
c) Inflation tax is imposed on the super-rich to subsidize the poor against the risks of inflation.
d) Inflation tax is imposed on those who strike and cause harm to public property in times of
high inflation.
Q.49) Solution (a)
Inflation tax is not an actual legal tax paid to a government; instead "inflation tax" refers to the
penalty for holding cash at a time of high inflation. When the government prints more money or
reduces interest rates, it floods the market with cash, which raises inflation in the long run. If an
investor is holding securities, real estate or other assets, the effect of inflation may be
negligible. If a person is holding cash, though, this cash is worth less after inflation has risen. The
degree of decrease in the value of cash is termed the inflation tax for the way it punishes people
who hold assets in cash, which tend to be lower- and middle-class wage earners.
Q.50) Which of the following is incorrect about Wholesale Price Index?
a) Wholesale Price Index (WPI) represents the price of goods at a wholesale stage i.e. goods
that are sold in bulk and traded between organizations instead of consumers.
b) The WPI has 3 categories namely Primary article group, fuel and power category and the
manufactured products category.
c) Primary articles have the highest weightage whereas the manufactured products have the
lowest weightage.
d) Indian WPI is published by the Economic Advisor, Ministry of Commerce and Industry.

www.iasbaba.com

25

Economics- Subject wise compilation of 60 Day Plan- 2016


Q.50) Solution (c)
WPI represents the price of goods at a wholesale stage i.e. goods that are sold in bulk and
traded between organizations instead of consumers.
Definition: Wholesale Price Index (WPI) represents the price of goods at a wholesale stage i.e.
goods that are sold in bulk and traded between organizations instead of consumers. WPI is used
as a measure of inflation in some economies.
Description: WPI is used as an important measure of inflation in India. Fiscal and monetary
policy changes are greatly influenced by changes in WPI. In the United States, Producer Price
Index (PPI) is used to measure inflation.
WPI is an easy and convenient method to calculate inflation. Inflation rate is the difference
between WPI calculated at the beginning and the end of a year. The percentage increase in WPI
over a year gives the rate of inflation for that year.
Q.51) Call and Term money markets are the sub-markets of Indian money market. Which of
the following statements are correct about these short term markets?
1. Call money refers to borrowing and lending of funds for one day.
2. Term money refers to borrowing and lending of funds for a period of more than 14 days.
3. Interest rates in these markets are fixed by banks.
4. Since banks work as both lenders and borrowers in these markets, they are also known as
Inter-Bank market.
Select the code from the following:
a) 1,2 and 3
b) 2,3 and 4
c) 1,2 and 4
d) All of the above
Q.51) Solution (c)
Call Money, Notice Money and Term Money markets are sub-markets of the Indian Money
Market. These refer to the markets for very short term funds.
Call Money refers to the borrowing or lending of funds for 1 day.
Notice Money refers to the borrowing and lending of funds for 2-14 days.
Term money refers to borrowing and lending of funds for a period of more than 14 days.
Interest rates in these markets are market determined i.e. by the demand and supply of short
term funds. In India, 80% demand comes from the public sector banks and rest 20% comes from
foreign and private sector banks.
Since banks work as both lenders and borrowers in these markets, they are also known as InterBank market. The short term fund market in India is located only in big commercial centres such

www.iasbaba.com

26

Economics- Subject wise compilation of 60 Day Plan- 2016


as Mumbai, Delhi, Chennai and Kolkata. The intervention of RBI is prominent in the short term
funds money market in India. Call Money / Notice Money market is most liquid money market
and is indicator of the day to day interest rates. If the call money rates fall, this means there is a
rise in the liquidity and vice versa.
Q.52) Consider the following statements:
1. Import of gold is inflationary
2. Providing subsidies through Direct Benefit Transfer causes demand pull inflation
3. International increase in crude prices will cause cost push inflation in Indian market
4. Inflow of FOREX causes inflation
Which of the above statements are correct?
a) 1 and 4
b) 1,2 and 3
c) 2,3 and 4
d) All of the above
Q.52) Solution (d)
With import of gold and inflow of FOREX, RBI has to print equivalent amount of currency. This
increases the liquidity and hence inflation increases.
International increase in crude oil causes increase in cost of production. This causes cost push
inflation.
DBT increases the money in consumers hand, causing demand pull inflation.
Q.53) Which of the following statements correctly explains the term Stagflation?
a) Persistent high inflation combined with high unemployment
b) Reduction of the general level of prices in the economy
c) Decrease in the rate of inflation
d) Constant prices of commodities for a long time in an economy
Q.53) Solution (a)
Stagflation is persistent high inflation combined with high unemployment and stagnant demand
in a country's economy.
Q.54) Which of the following economic entities is/are benefitted with inflation?
1. Debtors
2. Exporters
3. Savings bank account holders
4. Creditors
Select the code from the following:
a) 1 and 2
b) 1 only

www.iasbaba.com

27

Economics- Subject wise compilation of 60 Day Plan- 2016


c) 2,3 and 4
d) None of the above
Q. 54) Solution (b)
Debtors will be benefitted. As the market value of money is reduced, and the amount paid by a
debtor is fixed, he will be paying less value in reality.
Q.55) Consider the following statements regarding sugarcane and sugar pricing policy.
1. The concept of Statutory Minimum Price (SMP) of sugarcane was replaced with the Fair and
Remunerative Price (FRP)' of sugarcane.
2. Sugar is an essential commodity under the Essential Commodities Act, 1955.
3. The Central Government has been following a policy of partial control and dual pricing for
sugar.
Which of the above statements are correct?
a) 1 and 2
b) 2 and 3
c) 1 and 3
d) All of the above
Q.55) Solution (a)
Sugar is an essential commodity under the Essential Commodities Act, 1955. With the
amendment of the Sugarcane (Control) Order, 1966 on 22.10.2009, the concept of Statutory
Minimum Price (SMP) of sugarcane was replaced with the Fair and Remunerative Price (FRP) of
sugarcane for 2009-10 and subsequent sugar seasons.
The year 2013-14 was a water-shed for the sugar industry. The Central Government considered
the recommendations of the committee headed by Dr. C. Rangarajan on de-regulation of sugar
sector and decided to discontinue the system of levy obligations on mills for sugar produced
after September, 2012 and abolished the regulated release mechanism on open market sale of
sugar. The de-regulation of the sugar sector was undertaken to improve the financial health of
sugar mills, enhance cash flows, reduce inventory costs and also result in timely payments of
cane price to sugarcane farmers. The recommendations of the Committee relating to Cane Area
Reservation, Minimum Distance Criteria and adoption of the Cane Price Formula have been left
to State Governments for adoption and implementation, as considered appropriate by them
Q.56) Consider the following statements with respect to RBI
1. RBI or Reserve bank of India was established under the government of India act 1935
2. A palm tree and a Bengal tiger form the logo of RBI
Select the correct option
a) 1 only
b) 2 only

www.iasbaba.com

28

Economics- Subject wise compilation of 60 Day Plan- 2016


c) Both
d) None
Q.56) Solution (c)
https://www.rbi.org.in/currency/museum/m-ane.html
Now as per Laxmikanth, Chapter on Historical Background- RBI was established under the GoI
Act 1935 but as per RBI official website it was established under RBI act 1934. Which to select?
Now this is objectionable.
Q.57) Consider the following statements.
1. An extremely low aggregate demand.
2. Comparatively lower inflation.
3. Production houses go for forced labour cut in order to reduce production cost.
Which of the following stage of Business cycle best represents the above characteristics?
a) Recession
b) Recovery
c) Depression
d) Boom
Q.57) Solution (c)
The major traits of depression could be as given below:
(a) An extremely low aggregate demand in the economy causes activities to decelerate;
(b) The inflation being comparatively lower;
(c) The employment avenues start shrinking forcing unemployment rate to grow fast;
(d) To keep the business going, production houses go for forced labour-cuts or retrenchment
(to cut down the production cost and be competitive in the market) etc.
Recession Vs Depression:
Recession is somewhat similar to the phase of depression we may call it a mild form of
depression fatal for economies as this may lead to depression if not handled with care and in
time.
When a downturn in the GDP lasts for at least six months of time, it is termed as recession.
When the GDP drops by more than 10 percent, it results in depression.
Q.58) Consider the following statements with regards to inflation indexed bonds (IIBs).
1. IIBs will provide inflation protection to principal only.
2. Existing tax provisions will be applicable on interest payment and capital gains on IIBs.
3. Foreign institutional investors (FIIs) are not allowed to invest in IIBs.
Which of the above statements are NOT correct?
a) 2 and 3 only
b) 1 and 2 only

www.iasbaba.com

29

Economics- Subject wise compilation of 60 Day Plan- 2016


c) 1 and 3 only
d) All of the above
Q.58) Solution (c)
Inflation Indexed Bonds (IIBs) were issued in the name of Capital Indexed Bonds (CIBs) during
1997. The CIBs issued in 1997 provided inflation protection only to principal and not to interest
payment. IIBs will provide inflation protection to both principal and interest payments.
Tax provisions will be applicable on interest payment and capital gains on IIBs. There will be no
special tax treatment for these bonds.
IIBs would be Government securities (G-Sec) and the different classes of investors eligible to
invest in G-Secs would also be eligible to invest in IIBs. FIIs would be eligible to invest in the IIBs
but subject to the overall cap for their investment in G-Secs (currently USD 25 billion).
For more details:
https://rbi.org.in/scripts/FAQView.aspx?Id=91
Q. 59) Which of the following are correct w.r.t. Asset Reconstruction Companies?
1. It is set up to reconstruct or re-package assets to make them more saleable. The assets in
question here are loans from banks, card companies, financial institutions etc.
2. The word "asset reconstruction" in India was used in Narsimham I report where it was
envisaged for the setting up of a central Asset Reconstruction Fund with money contributed
by the Central Government.
3. ARCs focus on NPAs and allow the banking system to act as "clean bank".
Select the correct answer
a) 1 only
b) 2 only
c) 2 and 3 only
d) All of the above
Q.59) Solution (d)
ARC is in news for multiple reasons especially with NPAs being all time high. Hence we expect a
question. Here is the basic understanding about ARCs.
An Asset Reconstruction Company (ARC) is a company that is set up to do exactly what the
name suggests reconstruct or re-package assets to make them more saleable. The assets in
question here are loans from banks, card companies, financial institutions etc.
Why do we need ARCs? Bad loans are essentially of two types those that are a consequence
of routine banking operations and those that are a reflection of a greater systematic rot, as in
the Indian context where the bulk of non-performing assets (NPAs) are due to government
interference/loan waivers/difficulties in recovering dues etc. There are essentially two
approaches to tackling NPAs one, leave the banks to manage their own bad. Two, do the

www.iasbaba.com

30

Economics- Subject wise compilation of 60 Day Plan- 2016


same thing on a concerted, central level, through a centralized agency or agencies. ARCs are
centralised agencies for resolving bad loans created out of a systematic crisis. Non-performing
assets (NPAs) can be assigned to ARCs by banks at a discounted price, enabling a one-time
clearing of the balance sheet of banks of sticky loans.
At the same time, the ARC can float bonds and recover dues from the borrowers directly. ARCs
can have several alternate structures. They can either be publicly or privately owned or a
combination of both, and can be either separately capitalised units or wholly-owned
subsidiaries.
Q.60) Which of the following are correct about Monetary Policy?
1. The strategy to influence money supply and interest rates to affect output and inflation.
2. Balancing savings and investments is a major objective along with generating employment.
3. Exchange rate stabilization is achieved through monetary policy.
4. Monetary policy can be expansionary or contractionary depending on the status of the
macroeconomic parameters.
Select the correct answer
a) 1 and 3 only
b) 1 and 4 only
c) 3 and 4 only
d) All of the above
Q.60) Solution (d)
Monetary policy consists of the actions of a central bank, currency board or other regulatory
committee that determine the size and rate of growth of the money supply, which in turn
affects interest rates. Monetary policy is maintained through actions such as modifying the
interest rate, buying or selling government bonds, and changing the amount of money banks are
required to keep in the vault (bank reserves).
Further
reading:
http://www.yourarticlelibrary.com/policies/monetary-policy-meaningobjectives-and-instruments-of-monetary-policy/11134/
Q.61) Consider the following statements about Cash Reserve Ratio:
1. It is the ratio of deposits which banks have to keep with RBI as savings account.
2. Banks get a fixed interest on CRR from RBI.
3. With increase in CRR the lending capacity of banks will increase.
Which of the above statements are correct?
a) 1 and 2
b) 2 and 3
c) 1 and 3
d) None of the above

www.iasbaba.com

31

Economics- Subject wise compilation of 60 Day Plan- 2016


Q.61) Solution (d)
It is the ratio of Deposits which banks have to keep with RBI. Under CRR a certain percentage of
the total bank deposits has to be kept in the current account with RBI. Banks dont earn
anything on that.
Banks will not have access to this amount. They cannot use this money for any of their economic
or commercial activities. Banks cant lend this portion of money to corporate or individual
borrowers. With increase in CRR the lending capacity of banks will decrease as they will have
less money to lend.
Q.62) Which of the following is incorrect about Bank rate?
a) It is rate at which RBI lends long term to commercial banks.
b) It requires collateral security like in case of repo-reverse repo rate.
c) It is a penal rate and is aligned with MSF in 2011.
d) Lower bank rates can help to expand the economy, when unemployment is high, by
lowering the cost of funds for borrowers.
Q.62) Solution (b)
Bank rate being long term lending rate doesnt require collaterals like the G-secs in case of Repo
or MSF.
Bank rate is the rate charged by the central bank for lending funds to commercial banks. Bank
rates influence lending rates of commercial banks. Higher bank rate will translate to higher
lending rates by the banks. In order to curb liquidity, the central bank can resort to raising the
bank rate and vice versa
Further reading: https://www.rbi.org.in/scripts/FAQView.aspx?Id=51
Q.63) Consider the following about ready forward contracts.
1. Its a transaction where 2 parties agree to sell and repurchase the same security.
2. Repo rate is the rate at which the central bank of a country (Reserve Bank of India in case of
India) lends money to commercial banks in the event of any shortfall of funds.
3. The difference between Repo and Reverse repo rate is always 100 basis points with the
latter being lower.
4. The difference between Repo and MSF is always 100 basis points with the latter being
higher.
Select the correct answer:
a) 1 and 3 only
b) 1 and 2 only
c) 2 and 3 only
d) 1, 3 and 4 only

www.iasbaba.com

32

Economics- Subject wise compilation of 60 Day Plan- 2016


Q.63) Solution (b)
Repurchase Options or in short Repo, is a money market instrument, which enables
collateralised short term borrowing and lending through sale/purchase operations in debt
instruments.
"repo" means an instrument for borrowing funds by selling securities with an agreement to
repurchase the securities on a mutually agreed future date at an agreed price which includes
interest for the funds borrowed; "reverse repo" means an instrument for lending funds by
purchasing securities with an agreement to resell the securities on a mutually agreed future
date at an agreed price which includes interest for the funds lent."
This is the general definition of Repo and Reverse Repo in India. The securities transacted here
can be either government securities or corporate securities or any other securities which the
Central bank permits for transaction. Non-sovereign securities are used in many global markets
for repo operations. Unlike them, Indian repo market predominantly uses sovereign securities,
though repo is allowed on corporate bonds and debentures
The current repo, reverse repo and MSF are 6.5%, 6% and 7% respectively. Hence it is no bound
rule that there has to be a 100 basis points difference between them.
Q.64) Marginal standing facility (MSF) is a window for banks to borrow from the Reserve Bank
of India in an emergency situation when inter-bank liquidity dries up completely. Which of the
following is incorrect about MSF?
a) MSF is a penal rate as the repo limit is exhausted and also SLR limit is breached at times.
b) MSF operations also become necessary as the repo operations are limited to a specific
period of the day.
c) Only scheduled commercial banks can use this route with government securities including
SLR as collateral.
d) If banks do not have excess SLR it cannot borrow under MSF.
Q.64) Solution (d)
Marginal Standing Facility (MSF) is a new scheme announced by the Reserve Bank of India (RBI)
in its Monetary Policy (2011-12) and refers to the penal rate at which banks can borrow money
from the central bank over and above what is available to them through the LAF window.
MSF, being a penal rate, is always fixed above the repo rate. The MSF would be the last resort
for banks once they exhaust all borrowing options including the liquidity adjustment facility by
pledging government securities, where the rates are lower in comparison with the MSF. The
MSF would be a penal rate for banks and the banks can borrow funds by pledging government
securities within the limits of the statutory liquidity ratio. The scheme has been introduced by
RBI with the main aim of reducing volatility in the overnight lending rates in the inter-bank
market and to enable smooth monetary transmission in the financial system.

www.iasbaba.com

33

Economics- Subject wise compilation of 60 Day Plan- 2016


To balance the liquidity, RBI uses the sole independent "policy rate" which is the repo rate (in
the LAF window) and the MSF rate automatically gets adjusted to a fixed per cent above the
repo rate (MSF was originally intended to be 1% above the repo rate). MSF is at present aligned
with the Bank rate.
Even if the Banks do not have excess SLR they still can opt for MSF and it is allowed within
certain limits. The other statements are true.
Q.65) Consider the following about SLR. Which of the following are correct?
1. The ratio of liquid assets to net demand and time liabilities (NDTL) that they should keep in
designated liquid assets is called statutory liquidity ratio (SLR).
2. To control expansion of bank credit and ensure solvency of commercial banks is major
objective.
3. Banks hold public sector bonds, current account balances with other banks and gold as SLR.
4. Banks can also hold multilateral bonds and foreign exchange as part of SLR
Select the correct answer:
a) 1 only
b) 1 and 2 only
c) 1, 2 and 3 only
d) All of the above.
Q.65) Solution (c)
The Statutory Liquidity Ratio (SLR) is a prudential measure under which (as per the Banking
Regulations Act 1949) all Scheduled Commercial Banks in India must maintain an amount in one
of the following forms as a percentage of their total Demand and Time Liabilities (DTL) / Net DTL
(NDTL);
1. Cash.
2. Gold; or
3. Investments in un-encumbered Instruments that include;
(a) Treasury-Bills of the Government of India.
(b) Dated securities including those issued by the Government of India from time to time under
the market borrowings programme and the Market Stabilization Scheme (MSS).
(c) State Development Loans (SDLs) issued by State Governments under their market
borrowings programme.
(d) Other instruments as notified by the RBI.
Traditionally the amount to be held thus was stipulated to be no lower than 25 percent and not
exceeding 40 percent of the banks total DTL. However, effective from January, 2007 the floor of
25 percent on the SLR was removed following an amendment of the Banking Regulation Act,
1949.
Further reading: http://www.arthapedia.in/index.php?title=Statutory_Liquidity_Ratio
Q.66) Consider the following about SLR and CRR.

www.iasbaba.com

34

Economics- Subject wise compilation of 60 Day Plan- 2016


1. SLR restricts the banks leverage in pumping more money into the economy while CRR is the
portion of deposits that banks have to maintain within central bank.
2. CRR is allowed only in Cash and Government securities while SLR is allowed in multiple other
designated forms.
Select the correct answer:
a) 1 only
b) 2 only
c) Both 1 and 2
d) None of the above
Q.66) Solution (a)
The question tries to catch the aspirant on reading and attention to detail. CRR is allowed only in
Cash is a known fact.
Cash Reserve Ratio (CRR): Each bank has to keep a certain percentage of its total deposits with
RBI as cash reserves.
Statutory Liquidity Ratio (SLR): Amount of liquid assets such as precious metals(Gold) or other
approved securities, that a financial institution must maintain as reserves other than the cash.
Formula: SLR rate = (liquid assets / (demand + time liabilities)) 100%
Time liabilities are liabilities which the banks are liable to pay after a certain period of time. E.g.
A 1 year fixed deposit. Demand liabilities are liabilities which the banks are liable to pay on
being demanded by the customer. E.g. A savings account
CRR limits the ability of the banks to pump more money into the economy. SLR is used to limit
the expansion of bank credit, for ensuring the solvency of banks (even if all the loans by the
bank go bad, the bank can still retrieve a part of it by selling the gold/govt securities.
Further reading: http://www.livemint.com/Money/VLvbh3hAk9sSOCvYdqd7SI/DYK-Differencebetween-CRR-and-SLR.html
Q.67) Which of the following is incorrect about qualitative and quantitative methods used by
RBI to control credit supply?
a) Qualitative method controls the manner of channelizing cash and credit in the economy.
b) Qualitative method restricts credit for certain section and expands for others depending on
the erstwhile situation
c) Marginal requirement is increased for those business activities where the flow of credit is to
be restricted and hence a quantitative method.
d) Under rationing of credit RBI fixes ceiling of credit setting a limit to the loans and advances
that can be made to particular sector is a qualitative method.
Q.67) Solution (c)
Some of the methods employed by the RBI to control credit creation are: I. Quantitative Method
II. Qualitative Method.

www.iasbaba.com

35

Economics- Subject wise compilation of 60 Day Plan- 2016


The various methods employed by the RBI to control credit creation power of the commercial
banks can be classified in two groups, viz., quantitative controls and qualitative controls.
Quantitative controls are designed to regulate the volume of credit created by the banking
system qualitative measures or selective methods are designed to regulate the flow of credit in
specific uses.
Quantitative or traditional methods of credit control include banks rate policy, open market
operations and variable reserve ratio. Qualitative or selective methods of credit control include
regulation of margin requirement, credit rationing, regulation of consumer credit and direct
action.
Marginal requirement by the name sounds quantitative but is a qualitative method. So be
careful.
Further reading: http://www.yourarticlelibrary.com/banking/important-methods-adapted-byrbi-to-control-credit-creation/23490/
Q.68) Consider the following about Market Stabilization scheme.
1. MSS (Market Stabilisation Scheme) securities are issued with the objective of providing the
RBI with a stock of securities with which it can intervene in the market for managing
liquidity.
2. These securities are issued not to meet the government's expenditure.
3. The amounts raised under the MSS will be held in a separate identifiable cash account titled
the Market Stabilization Scheme Account (MSS Account) to be maintained and operated by
the RBI.
Select the correct answer
a) 1 only
b) 2 only
c) 1 and 3 only
d) All of the above
Q.68) Solution (d)
This scheme came into existence following a MoU between the Reserve Bank of India (RBI) and
the Government of India (GoI) with the primary aim of aiding the sterilization operations of the
RBI. Historically, the RBI had been sterilizing the effects of significant capital inflows on domestic
liquidity by offloading parts of the stock of Government Securities held by it. It is pertinent to
recall, in this context, that the assets side of the RBIs Balance Sheet (July 1 to June 30) includes
Foreign Exchange Reserves and Government Securities while liabilities are primarily in the form
of High Powered Money (consisting of Currency with the public and Reserves held in the RBI by
the Banking System). Thus, any rise in Foreign Exchange Reserves resulting from the
intervention of the RBI in the Foreign Exchange Markets (with the intention, say, to maintain the
exchange rate on the face of huge capital inflows) entails a corresponding rise in High Powered
Money. The Money Supply in the economy is linked to High Powered Money via the money
multiplier. Therefore, on the face of large capital inflows, to keep the liabilities side constant so

www.iasbaba.com

36

Economics- Subject wise compilation of 60 Day Plan- 2016


as to not raise the Supply of Money, corresponding reduction in the stock of Government
Securities by the RBI is necessary.
Q.69) Consider the following about fixed and floating interest rates.
1. A floating interest rate is an interest rate that is allowed to move up and down with the rest
of the market or along with an index but linked to an underlying benchmark rate.
2. A fixed interest rate is an interest rate on a liability, such as a loan or mortgage, that remains
fixed either for the entire term of the loan or for part of this term.
Select the correct answer:
a) 1 only
b) 2 only
c) Both 1 and 2
d) None of the above
Q.69) Solution (c)
A floating interest rate, also known as a variable or adjustable rate, refers to any type of debt
instrument, such as a loan, bond, mortgage, or credit, that does not have a fixed rate of interest
over the life of the instrument.
Floating interest rates typically change based on a reference rate (a benchmark of any financial
factor, such as the Consumer Price Index). One of the most common reference rates to use as
the basis for applying floating interest rates is the London Inter-bank Offered Rate, or LIBOR (the
rates at which large banks lend to each other)
A fixed interest rate is an interest rate on a liability, such as a loan or mortgage, that remains
fixed either for the entire term of the loan or for part of this term. A fixed interest rate may be
attractive to a borrower who feels that the interest rate might rise over the term of the loan,
which would increase his or her interest expense. A fixed interest rate, therefore, avoids the
interest rate risk that comes with a floating or variable interest rate, wherein the interest rate
payable on a debt obligation depends on a benchmark interest rate or index. The UPSC trend of
asking questions is coming down to fundamentals of the current and trending topics. This is
relevant w.r.t Chinas devaluation and after effects.
Q.70) The Reserve Bank of India (RBI) constituted and Expert Committee to Revise and
Strengthen the Monetary Policy Framework under the Chairmanship of Dr. Urjit R.Patel.
Consider the following.
1. It has suggested that the apex bank should adopt the new CPI (consumer price index) as the
measure of the nominal anchor for policy communication.
2. The committee asked the Central Government to ensure that the fiscal deficit as a ratio to
GDP (gross domestic product) is brought down to 3.0 per cent by 2016-17.
3. The Patel panel felt that the monetary policy decision-making should be vested with a
monetary policy committee (MPC) which has participation of both government and RBI.

www.iasbaba.com

37

Economics- Subject wise compilation of 60 Day Plan- 2016


4. The FSLRC under Justice (retd) B N Srikrishna had also suggested for the formation of an
MPC.
Select the correct answer:
a) 1 and 2 only
b) 1 and 3 only
c) 1, 2 and 3 only
d) 1, 2 and 4 only
Q.70) Solution (d)
Urjit patel Committee is avery important committee of RBI. It suggested only RBI members in
the MPC with no government nominees. This was point of conflict even.
Further
reading:
http://www.thehindu.com/business/Economy/urjit-panel-suggests-4-cpiinflation-target/article5602626.ece
https://rbi.org.in/Scripts/PublicationReportDetails.aspx?ID=743
Q.71) What is common for all the following?
1. S S Tarapore Committee of 2006
2. Percy Mistry Committee on 2007
3. Jahangir Aziz working group of 2008
4. Dr. Raghuram Rajan Committee of 2009
5. Justice B N Srikrishnas FSLRC of 2013
Select the correct answer.
a) Capital and Current account convertibility.
b) Poverty alleviation
c) Public Debt management Agency
d) Categorisation of states for discretionary grants.
Q.71) Solution (c)
There were multiple committees that suggested PDMA. Especially with the agency formation
becoming point of tussle between Ministry of Finance and RBI, we expect a question this time.
These committees have been reffered in government documents repeatedly especially post
2015 budget.
Further reading:
http://www.arthapedia.in/index.php?title=Public_Debt_Management_Agency_(PDMA)
Q.72) Which of the following is incorrect about Public Debt Management Agency the
government intends to set up?
a) Public Debt Management Agency (PDMA) is a specialized independent agency that manages
the internal and external liabilities of the Central Government in a holistic manner and
advises on such matters in return for a fee.

www.iasbaba.com

38

Economics- Subject wise compilation of 60 Day Plan- 2016


b) PDMA is considered to be set up with the objective of "maximising the cost of raising and
servicing public debt over the long-term within an acceptable level of risk at all times, under
the general superintendence of the central government".
c) An autonomous PDMA can be the catalyst for wider institutional reform, including building a
government securities market, and bring in transparency about public debt.
d) Genesis of the thinking on an independent debt management office is traced back to the
Committee on Capital Account Convertibility (1997) and the Review Group of Standing
Committee on International Financial Standards & Codes (2004).
Q. 72) Solution (b)
Public Debt Management Agency (PDMA) is a specialized independent agency that manages the
internal and external liabilities of the Central Government in a holistic manner and advises on
such matters in return for a fee. In other words, PDMA is the Investment Banker or Merchant
Banker to the Government. PDMA manages the issue, reissue and trading of Government
securities, manages and advises the Central Government on its contingent liabilities and
undertakes cash management for the central government including issuing and redeeming of
short term securities and advising on its cash management.
PDMA is considered to be set up with the objective of "minimising the cost of raising and
servicing public debt over the long-term within an acceptable level of risk at all times, under the
general superintendence of the central government". This will guide all of its key functions,
which include managing the public debt, cash and contingent liabilities of Central Government,
and related activities.
Further reading:
http://www.arthapedia.in/index.php?title=Public_Debt_Management_Agency_(PDMA)
Q.73) Which of the following is not correctly matched?
a) Lakadwala Poverty estimation
b) Justice R V Eshwar Taxation
c) Kirit Parikh Disinvestment
d) Urjit patel Monetary Policy
Q.73) Solution (c)
The Kirit Parikh panel was set up by the Petroleum and Natural Gas Ministry to suggest a
methodology for pricing of diesel and cooking fuel.
The panel has said that the Government should take steps to pass on the impact of rise in price
of diesel to consumers, and move rapidly towards making the price of diesel marketdetermined. The expert group recommends that in view of high under-recovery of Rs. 10.51
per litre on diesel, HSD prices be raised by Rs. 5 per litre with immediate effect. The balance
under-recovery should be made up through a subsidy of Rs. 6 per litre to public sector oil
marketing companies (OMCs). The subsidy on diesel should be capped at Rs. 6 per litre litre. This
will imply freeing of price of diesel beyond this cap, the panel has said.

www.iasbaba.com

39

Economics- Subject wise compilation of 60 Day Plan- 2016


Q.74) Consider the following about Phillips Curve.
1. The Phillips curve represents the relationship between the rate of inflation and the
unemployment rate.
2. Phillips found a consistent inverse relationship: when unemployment was high, wages
increased slowly; when unemployment was low, wages rose rapidly.
3. It showed the rate of wage inflation that would result if a particular level of unemployment
persisted for some time.
4. Price stability has a trade-off against unemployment and some level of inflation could be
considered desirable in order to minimize unemployment, is the core argument.
Select the correct answer:
a) 1 and 2 only
b) 1 and 4 only
c) 2, 3 and 4 only
d) 1, 2, 3 and 4 only
Q.74) Solution (d)
The Phillips curve is an economic concept developed by A. W. Phillips stating that inflation and
unemployment have a stable and inverse relationship. According to the Phillips curve, the lower
an economy's rate of unemployment, the more rapidly wages paid to labor increase in that
economy.
Further reading: http://www.econlib.org/library/Enc/PhillipsCurve.html
Q.75) Consider the following.
1. Deflation is used to describe instances when the inflation rate has reduced marginally over
the short term
2. Disinflation is a general decline in prices, often caused by a reduction in the supply of money
or credit.
Select the correct answer:
a) 1 only
b) 2 only
c) Both 1 and 2
d) None of the above
Q.75) Solution (d)
Deflation and disinflation are used with reference to change in general price levels in an
economy. While these two terms sound similar, they have very different meanings. As we know,
inflation refers to the rise in price levels in an economy, and deflation is the opposite, a fall in
price levels. Disinflation, on the other hand, refers to a slower rate of inflation.
Further reading: http://www.livemint.com/Money/rmR2KGnzn18ltvP6zBgpsO/Dejargoneddeflation-and-disinflation.html

www.iasbaba.com

40

Economics- Subject wise compilation of 60 Day Plan- 2016

Q.76) If deflation is general decline in prices caused by a reduction in the supply of money
then which of the following is not the possible consequence of the same?
a) Demand from businesses and consumers to buy products falls
b) Employment growth rate stabilizes and is upward in trend
c) Debt servicing becomes more expensive
d) Governments can resort to tax cuts to boost demand from consumers and businesses
Q.76) Solution (b)
Deflation is when prices actually fall. Such a situation does seem favorable but in reality its not
a position that any economy would like to be in. Deflation usually moves hand in hand with
economic slowdown, lower productivity and loss of jobs. Just like how inflation decreases the
value of money, deflation increases its value. This incentivizes people to save money to buy later
when goods are cheaper, which, in turn, leads to further slowing of economic growth. During
deflation, value of money increases and goods are cheaper but you may end up earning less
thanks to slowing growth.
During the Great Depression, there was severe deflation; it had moved into double digits. Japan
has been struggling with deflation for the past two decades. Only recently did its Prime Minister,
Shinzo Abe, declare that the period of deflation has come to an end and that economic recovery
is on its way.
A positive impact of deflation is increased export competitiveness as most other economies are
in an inflationary trend. Japan has benefited from competitive exports in the past decade or so.
Q.77) Consider the following about inflation.
1. When the government doesnt attempt to curb price rise allowing the free market
mechanism to function it is said to be open inflation.
2. When international bodies interrupts a price rise in a country through price control
measures and subsidies it is said to be internationally controlled inflation.
Select the correct answer:
a) 1 only
b) 2 only
c) Both 1 and 2
d) None of the above
Q.77) Solution (a)
Inflation is often open and suppressed. Inflation is open when markets for goods or factors of
production are allowed to function freely, setting prices of goods and factors without normal
interference by the authorities. Thus open inflation is the result of the uninterrupted operation
of the market mechanism.

www.iasbaba.com

41

Economics- Subject wise compilation of 60 Day Plan- 2016


There are no checks or controls on the distribution of commodities by the government. Increase
in demand and shortage of supplies persists which tend to lead to open inflation. Unchecked
open inflation ultimately leads to hyper-inflation.
Suppressed Inflation: On the contrary when the government imposes physical and monetary
controls to check open inflation, it is known as repressed or suppressed inflation. The market
mechanism is not allowed to function normally by the use of licensing, price controls and
rationing in order to suppress extensive rise in prices.
Q.78) Suppressed inflation adversely affects the economy. What are the probable
consequences?
1. When the distribution of commodities is controlled, the prices of uncontrolled commodities
rise very high.
2. Suppressed inflation reduces the incentive to work because people do not get the
commodities which they want to have.
3. Suppressed inflation leads to black marketing, corruption, hoarding and profiteering. It
invites extralegal powers of control.
Select the correct answer:
a) 1 and 2 only
b) 1 and 3 only
c) 2 and 3 only
d) 1, 2 and 3 only
Q.78) Solution (d)
Suppressed inflation results when efforts are made to increase domestic production and reduce
import demand by tariffs, import restrictions, limits on foreign loans, voluntary import
agreements, etc. So long as such controls exist, the present demand is postponed and there is
diversion of demand from controlled to uncontrolled commodities. But as soon as these
controls are removed, there is open inflation. Its Effects:
(1) When the distribution of commodities is controlled, the prices of uncontrolled commodities
rise very high.
(2) Suppressed inflation reduces the incentive to work because people do not get the
commodities which they want to have.
(3) Controlled distribution of goods also leads to misallocation of resources. This results in the
diversion of productive resources from essential to non-essential industries.
(4) Frictions increase in the labour market when high inflation is associated with higher
unemployment.
(5) Suppressed inflation leads to black marketing, corruption, hoarding and profiteering. It
invites extralegal powers of control.
Q.79) The Nairobi Package was adopted at the WTO's Tenth Ministerial Conference, held in
Nairobi, Kenya. It contains a series of Ministerial Decisions on

www.iasbaba.com

42

Economics- Subject wise compilation of 60 Day Plan- 2016


1.
2.
3.
4.
5.

Special Safeguard Mechanism for Developed Countries


Issues of Least Developed Countries
Export Competition
Export of Cotton, Pulses and Rice and its by products
Public Stockholdings

Select the correct code


a) 1, 2, 3 and 5
b) 2, 3, 4 and 5
c) 1, 2, 3 and 4
d) 2, 3 and 5
Q.79) Solution (d)
The Nairobi Package was adopted at the WTO's Tenth Ministerial Conference, held in Nairobi,
Kenya, from 15 to 19 December 2015. It contains a series of six Ministerial Decisions on
agriculture, cotton and issues related to least-developed countries (LDCs). A Ministerial
Declaration outlining the Package and the future work of the WTO was adopted at the end of
the five-day Conference.
https://www.wto.org/english/thewto_e/minist_e/mc10_e/nairobipackage_e.htm
http://www.firstpost.com/business/nothing-at-nairobi-wto-ministerial-leaves-india-anddeveloping-countries-in-the-lurch-2553428.html
http://www.thehindu.com/business/Industry/wto-nairobi-meeting-government-to-respond-towtos-nairobi-package-in-parliament/article8011127.ece
Q.80) According to RBI guidelines, Basel III norms are to be implemented by all banks till 2019.
Consider the following statements:
1. Capital to Risk weighted Assets Ratio (CRAR) is the percentage of banks risk weighted credit
exposures.
2. RBI has fixed CRAR as 9%.
3. Capital Adequacy Ratio (CAR) is expressed as CRAR.
4. Higher the CAR, higher is the risk.
Which of the above statements are correct?
a)
b)
c)
d)

1,2 and 3
2,3 and 4
1,3 and 4
All of the above

Q.80) Solution (a)


The capital adequacy ratio (CAR) is a measure of a bank's capital. It is expressed as a percentage
of a bank's risk weighted credit exposures. Also known as capital-to-risk weighted assets ratio

www.iasbaba.com

43

Economics- Subject wise compilation of 60 Day Plan- 2016


(CRAR), it is used to protect depositors and promote the stability and efficiency of financial
systems around the world.
In simple terms Capital Adequacy Ratio is Capital/risk. Lower the CAR, higher is the risk.
Q.81) Consider the following statements:
1. When Repo Rate increases, borrowing from RBI becomes more expensive.
2. Increase in Reverse Repo decreases the liquidity in the market.
3. Reverse Repo is fixed .5 percent points more than Repo.
Which of the above statements are correct?
a) 1 and 2
b) 2 and 3
c) 1 and 3
d) All of the above
Q.81) Solution (a)
Reverse Repo is fixed .5 percentage points less than Repo.
Q.82) Which of the following is incorrectly matched?
a) Tax administration reforms commission Justice Ashok Kumar Mathur
b) Expenditure Management Commission Bimal Jalan
c) Committee to simplify Income Tax laws Justice R V Easwar
d) FRBM review panel N K Singh
Q.82) Solution (a)
Tax administration reforms commission is headed by Sri Parthasarathi Shome and not Justic A K
Mathur. Justice Mathur is head of 7th Pay Commission.
Q.83) The GST Council will be the body that decides which taxes levied by the Centre, States
and local bodies will go into the GST. Who among the following are members of GST Council?
1. Union Minister of Finance
2. Minister of Finance of each state
3. Union Minister of state for Finance
4. Chief Economic Advisor to Minister of Finance
5. Governor of RBI
Select the correct answer
a) 1, 2 and 4 only
b) 1, 2, and 3 only
c) 1, 2, 3 and 5 only
d) All of the above
Q.83) Solution (b)

www.iasbaba.com

44

Economics- Subject wise compilation of 60 Day Plan- 2016


The 122nd Constitution Amendment Bill seeks to establish a GST Council tasked with optimising
tax collection for goods and services by the State and Centre. The Council will consist of the
Union Finance Minister (as Chairman), the Union Minister of State in charge of revenue or
Finance, and the Minister in charge of Finance or Taxation or any other, nominated by each
State government.
The GST Council will be the body that decides which taxes levied by the Centre, States and local
bodies will go into the GST; which goods and services will be subjected to GST; and the basis and
the rates at which GST will be applied.
Q.84) Consider the following about Gold Monetization Scheme.
1. The designated banks will accept gold deposits under the Short Term (1-3 years) Bank
Deposit as well as Medium (5-7 years) and Long (12-15 years) Term Government Deposit
Schemes.
2. Both principal and interest to be paid to the depositors of gold, will be valued in gold.
3. Customer will have the choice to take cash or gold on redemption, but the preference has to
be stated at the time of deposit.
4. The interest rate is decided by the banks concerned.
Select the correct answer
a) 1 and 2 only
b) 1, 2 and 3 only
c) 1, 3 and 4 only
d) All of the above
Q.84) Solution (d)
Its an important scheme of the government and is expected as a question. All the above are
right. Yes interest rates are decided by the banks.
Further reading: http://finmin.nic.in/swarnabharat/gold-monetisation.html

Doubts reg Q. 3) Gold Monetisation Scheme:


As per Econ.c Survey and SBI website:
Option for Redemption in gold or cash is available only for ST deposits.
For MT and LT deposits, Redemption is only in cash.
So, shouldn't statement 3 be wrong?
Statement 4 is also wrong - Interest rates on short-term (1-3 years) is decided by Banks but
the Interest rates on medium-term (5-7 years) and long-term (12-15 years) is decided by
RBI.
Clarification: The policy was updated and this clause of gold only for short term and cash
only for medium and long term was included.
www.iasbaba.com

45

Economics- Subject wise compilation of 60 Day Plan- 2016

Q.85) Which of following qualify as measures taken to curb tax evasion and black money by
government?
1. Income Disclosure Scheme of 2016
2. Amendment to DTAA with Mauritius
3. Place of Effective Management Rule to applicable
4. Undisclosed foreign income and assets act 2015
Select the correct answer
a) 1, 2 and 3 only
b) 1, 2 and 4 only
c) 1, 3 and 4 only
d) All of the above
Q.85) Solution (d)
All the above are right and crucial measures taken up by the government in last 2 financial
years. Place of Effective Management Rule may be raising eyebrows. So let us understand it.
http://www2.deloitte.com/in/en/pages/tax/articles/place-of-effective-management.html
Q.86) Consider the following.
1. Controller General of Accounts is part of Department of Revenue and prepares the
appropriation bill.
2. CGA is head of Indian Civil Accounts Service and administers it.
Select the correct answer:
a) 1 only
b) 2 only
c) Both 1 and 2
d) None of the above
Q.86) Solution (b)
No it is not Department of Revenue but Expenditure. Now that is nail biting. Isnt it?
Indian Civil Accounts Organisation performs a key role in delivery of financial management
services for the Government of India. The organisation provides payment services, supports the
tax collection system, performs government wide accounting, financial reporting functions,
preparation of budget estimates and carries out Internal Audit in civil ministries of the Union
Government. Controller General of Accounts (CGA) in Ministry of Finance heads the
organisation and is responsible for administering this system.
Q.87) Consider the following.
1. Ideally custom duties should be high on raw material import and low on export for a country
like India.

www.iasbaba.com

46

Economics- Subject wise compilation of 60 Day Plan- 2016


2. Similarly custom duties should be high on finished goods exports and low on import in line
with Make in India initiative.
Select the correct answer:
a) 1 only
b) 2 only
c) Both 1 and 2
d) None of the above
Q.87) Solution (d)
Just the reverse of both statements isnt it? Ideally custom duties should be low on raw material
import and high on export for a country like India to encourage manufacturing within India.
Q.88) Consider the following.
1. The Krishi Kalyan Cess and Swachch Bharat Cess are applicable on RBI and Postal services
but not on government insurance schemes for social sector.
2. Krishi Kalyan Cess and Swachch Bharat Cess proceeds are part of Consolidated Fund of India
and will be part of the shareable tax pool as per the recommendation of the Tax
Administration reform Council 2015-16.
Select the correct answer:
a) 1 only
b) 2 only
c) Both 1 and 2
d) None of the above
Q.88) Solution (d)
We know any Cess is not part of sharable tax pool and there has been no such recommendation.
17 types of services including RBI, Postal Education, insurance have been kept out of the above
service tax net.
Q.89) Consider the following about revenue and capital receipts.
1. Interest receipts, dividends and profits are parts of revenue receipts.
2. Article 112 of the constitution mandates expenditure is shown in revenue and other
categories.
3. Plan- Non Plan expenditure segregation has been constitutionally mandated and needs an
amendment to be removed.
4. Disinvestment proceeds are part of capital receipts.
Select the correct answer
a) 1 and 2 only
b) 1, 2 and 3 only
c) 1, 2 and 4 only

www.iasbaba.com

47

Economics- Subject wise compilation of 60 Day Plan- 2016


d) All of the above
Q.89) Solution (c)
Non plan expenditure is not a constitutional term. Rest should be self explainatory.
Further reading: http://pib.nic.in/budget2015/book/Budgetary.pdf
Q.90) Which of the following is incorrect w.r.t Ways and Means Advances (WMAs)?
a) RBI provides 2 types of WMAs to state governments one with and another without
collaterals.
b) WMAs given by RBI to government of India do not require any collateral.
c) Replacement of adhoc bills with WMAs represents an advance in fiscal discipline and
harmonization of fiscal and monetary policies.
d) WMAs are made at Reverse Repo rate.
Q.90) Solution (d)
The interest rate charged on WMA and overdrafts at present are the Bank Rate (9 per cent) and
the Bank Rate plus two percentage points (11 per cent), respectively.
Further reading: https://www.rbi.org.in/Scripts/PublicationReportDetails.aspx?ID=6

Doubts: ques 9- WMA are made at the repo rate


kindly clarify! question 9 ka d option is wrong becoz link provided in solution is 1999 ka
d option is correct becoz ways and means intrest is linked to repo rate only
Clarifications: Statement (d) is corrected
For more, refer https://rbi.org.in/scripts/BS_PressReleaseDisplay.aspx?prid=36110

Q.91) Which of the following is correct debt?


a) The union government borrows directly from international capital markets with the help of
debt management agencies.
b) Small savings and provident funds are part of other liabilities.
c) Dated securities as part of market borrowings are issued by union governments as T-bills
with 14 days, 91days, 182days maturity.
d) Public debt means only internal debt and other liabilities but not external debt.
Q.91) Solution (b)
The union government doesnt borrow directly from international capital markets. 14 day
treasury bills are only issued by state governments. Public debt includes both internal and
external debt.
www.iasbaba.com

48

Economics- Subject wise compilation of 60 Day Plan- 2016


Source: India Year Book Chapter : Finance

*Correction: In explanation, we had given 14 day treasury bills are only issued by state
governments. Rather 14 day bills are issued by the RBI to state governments and nobody
else... So it is not by state government but to state government... Please make the change.

Q.92) Which of the following recommendations of 14th Finance Commission regarding tax
Devolution are accepted by the Government?
1. StatesShare in the net proceeds of Union tax revenues increased to 42% from 32% earlier.
2. Sharing pattern under various CSS to undergo a change, with States sharing higher fiscal
responsibility for scheme implementation.
3. Distribution of grants to States for local bodies based on 2011 population data (90%weight)
and area (10%weight).
4. Revenue compensation to States under GST should be for five years;100% in first three
years, 75% in fourth year and 50% in fifth year.
Select the code from the following:
a) 1,2 and 3
b) 2,3 and 4
c) 1,3 and 4
d) All of the above
Q.92) Solution (a)
The last recommendation has not been accepted. It is still under consideration. Refer: 14th
Finance Commission.
Q.93) Consider the following statements regarding Financial Stability:
1. Financial stability could be defined as a situation in which the financial sector provides
critical services to the real economy without any discontinuity.
2. Financial Stability report is published every year by Ministry of Finance.
3. Financial stability is an important aspect of the Monetary Policy of India.
Which of the above statements are correct?
a) 1 and 2
b) 2 and 3
c) 1 and 3
d) All of the above
Q.93) Solution (c)

www.iasbaba.com

49

Economics- Subject wise compilation of 60 Day Plan- 2016


Financial stability means financial institutions individually and collectively are being able to
deliver their functions properly, withstanding external shocks and avoiding internal weaknesses.
India Financial Stability Report published by the Reserve Bank of India (March 2010), defines
financial stability: From a macro prudential perspective, financial stability could be defined as a
situation in which the financial sector provides critical services to the real economy without any
discontinuity.
During the time of the global financial crisis, RBI has made many unconventional measures to
protect the banking system. Liquidity support was provided abundantly so that no banks should
face stress. The RBI since 2010 is publishing India Financial Stability Report to assess financial
stability scenario in the country. Financial stability is now one of the three important objectives
of monetary policy besides price stability and credit support.
Q.94) Volume two of Economic Survey talks about a Chakravyuh problem of Indian economy.
Which of the following does Chakravyuh refers to?
a) Vicious cycle of poverty and lack of capital
b) Over dependence of economy on Foreign Direct investment
c) Lack of clear exit policy for companies
d) Conflict between economic growth and inflation
Q.94) Solution (c)
The Chapter 2 titled Chakravyuha Challenge of the Indian Economy.
Q.95) Consider the following Statements regarding Gyan Sangam summit:
1. It was a meeting organized by Ministry of Finance and RBI with central banks of Tax haven
countries to find a method to recover black money which is tapped outside.
2. It is a forum where the highest officials from public sector banks, the government and the
Reserve Bank of India, meet to discuss issues faced by the sector.
3. Focus was discussion on bank consolidation, NPAs and carrying out changes to recovery
laws.
Which of the above statements are correct?
a) 1 only
b) 2 and 3
c) 1 and 3
d) None of the above
Q.95) Solution (b)
Second Edition of Gyan Sangam was held this year to discuss consolidation of banks, NPAs,
changes to recovery laws and giving out employee stock options.

www.iasbaba.com

50

Economics- Subject wise compilation of 60 Day Plan- 2016


Q.96) SARFAESI Act refers to Securitisation and Reconstruction of Financial Assets and
Enforcement of Security Interest Act, 2002. Banks utilize this act as an effective tool for bad
loans (NPA) recovery. Which of the following statements are correct about this act?
1. Upon loan default, banks can seize the securities (except agricultural land) without
intervention of the court.
2. Court intervention is not required only in case of secured loans where bank can enforce the
underlying security eg hypothecation, pledge and mortgages.
3. If the asset in question is an unsecured asset, the bank would have to move the court to file
civil case against the defaulters.
Select the code from the following:
a) 1 and 2
b) 2 and 3
c) 1 and 3
d) All of the above
Q.96) Solution (d)
SARFAESI Act as we know is Securitisation and Reconstruction of Financial Assets and
Enforcement of Security Interest Act, 2002. Banks utilize this act as an effective tool for bad
loans ( NPA) recovery.
It is possible where non-performing assets are backed by securities charged to the Bank by way
of hypothecation or mortgage or assignment. Upon loan default, banks can seize the securities
(except agricultural land) without intervention of the court.
SARFAESI is effective only for secured loans where bank can enforce the underlying security eg
hypothecation, pledge and mortgages. In such cases, court intervention is not necessary, unless
the security is invalid or fraudulent.
However, if the asset in question is an unsecured asset, the bank would have to move the court
to file civil case against the defaulters.
Q.97) Which of the following regarding Tax Buoyancy are correct:
1. Tax buoyancy explains this relationship between the changes in governments tax revenue
growth and the changes in GDP.
2. When a tax is buoyant, its revenue increases without increasing the tax rate
Select the code from below:
a) 1 only
b) 2 only
c) Both 1 and 2
d) Neither 1 nor 2
Q.97) Solution (c)

www.iasbaba.com

51

Economics- Subject wise compilation of 60 Day Plan- 2016


There is a strong connection between the governments tax revenue earnings and economic
growth. The simple fact is that as the economy achieves faster growth, the tax revenue of the
government also goes up.
Tax buoyancy explains this relationship between the changes in governments tax revenue
growth and the changes in GDP. It refers to the responsiveness of tax revenue growth to
changes in GDP. When a tax is buoyant, its revenue increases without increasing the tax rate.
Q.98) Which of the following statements are correct regarding Pigovian Tax?
1. A Pigovian tax is a tax levied on any market activity that generates negative externalities.
2. It is often levied on companies that pollute the environment or create excess social costs.
3. A type of a Pigovian tax is a "sin tax", which is a special tax on tobacco products and alcohol.
Select the code from the following:
a) 1 and 2
b) 2 and 3
c) 1 and 3
d) All of the above
Q.98) Solution (d)
A Pigovian tax is a special tax that is often levied on companies that pollute the environment or
create excess social costs, called negative externalities, through business practices. In a true
market economy, a Pigovian tax is the most efficient and effective way to correct negative
externalities. A type of a Pigovian tax is a "sin tax", which is a special tax on tobacco products
and alcohol.
Q.99) NITI Aayog has replaced the older planning commission. Which of the following points
differentiates NITI aayog from Planning Commission?
1. NITI Aayog is an advisory body while Planning Commissions recommendations were
binding.
2. State representatives were not there in Planning Commission while NITI Aayog has included
Chief Ministers of all states as well as administrators of UTs.
3. The provision of regional council is there in Niti Ayog to address local / regional
development issues.
Select the code from the following:
a) 1 and 2
b) 2 and 3
c) 1 and 3
d) All of the above
Q.99) Solution (b)
Both NITI Aayog and Planning Commission were advisory bodies.

www.iasbaba.com

52

Economics- Subject wise compilation of 60 Day Plan- 2016


Q.100) Consider the following statements regarding subsidies in current fiscal year:
1. Subsidies on food, fertiliser and petroleum have been pegged higher by over 4 per cent.
2. The deregulation of diesel price, along with the introduction of direct benefit (subsidy)
transfer into the bank accounts of domestic LPG consumers, coupled with a sharp decline in
global crude oil prices has helped containing the petroleum subsidy bill.
Which of the above statements are correct?
a) 1 only
b) 2 only
c) Both 1 and 2
d) Neither 1 nor 2
Q.100) Solution (b)
Subsidies on food, fertiliser and petroleum have been pegged lower by over 4 per cent. This is
one of the biggest drop in subsidy expenditure. (Economic Survey- Public Finance).
Q.101) Which of the following statement are the recommendation of Tax Administration
Reforms Commission?
1. A minimum of 10% of the tax administrations budget must be spent on taxpayer services.
2. Pre-filled tax returns should be provided to all individuals. The taxpayer will have the option
of accepting the tax return or modifying it.
3. CBDT and CBEC should be fully integrated under Central Board of Direct and Indirect Taxes.
4. The Permanent Account Number (PAN) should be developed as a Common Business
Identification Number (CBIN), to be used by other departments such as customs, excise, etc.
Select the code from the following:
a) 1,3 and 4
b) 3 and 4
c) 1,2 and 3
d) All of the above
Q.101) Solution (d)
TARC made the following recommendations.
Consumer Focus:
There should be a separate vertical for delivery of taxpayer services in each Board. A
minimum of 10% of the tax administrations budget must be spent on taxpayer services.
The decision of the Ombudsman with regard to redressing taxpayer grievances should be
binding on tax officers.
Pre-filled tax returns should be provided to all individuals. The taxpayer will have the option
of accepting the tax return or modifying it.
Structure and Governance:

www.iasbaba.com

53

Economics- Subject wise compilation of 60 Day Plan- 2016

CBDT and CBEC should be fully integrated in 10 years. Within the next 5 years, they should
move towards a unified management structure under the Central Board of Direct and
Indirect Taxes.
The post of Revenue Secretary should be abolished and its functions should be assigned to
the two Boards. A Governing Council to oversee the working of the two Boards, and a Tax
Council to suggest policy and legislation should be set up.
Human Resource Development:
There should be a focus on specialisation, including lateral entry of specialists in the Boards.
Indian Revenue Service (IRS) officers should specialise in a particular tax administration
areas.
The Central Vigilance Commission should have a Member who has been an IRS officer. The
policy of not taking cognizance of anonymous complaints should be strictly followed.
Dispute Resolution and Management:
Retrospective legislation should be avoided.
Both Boards should start a special drive for review and liquidation of cases currently
clogging the system by setting up dedicated task forces.
A separate dispute management vertical should be set up in each Board. In addition, the
process of pre-dispute consultation before issuing a tax demand notice should be put into
practice.
Internal Processes: The Permanent Account Number (PAN) should be developed as a Common
Business Identification Number (CBIN), to be used by other departments such as customs,
excise, etc.
Q.102) Which of the following can be the objectives behind setting up Public Sector
Undertakings (PSUs)?
1. To build a self reliant economy
2. To prevent or reduce concentration of private economic power
3. Establish sound economic infrastructure
4. Assist in ancillarisation and spread the benefits of industrialisation.
Select the correct answer
a) 1, 2 and 3 only
b) 1, 2 and 4 only
c) 1, 3 and 4 only
d) All of the above
Q.102) Solution (d)
All the above are objectives along with following
1. To promote rapid economic development by filing critical gaps in the industrial structure
2. To provide basic infra-structural facilities for the growth of the economy;
3. To undertake economic activity strategically important for the growth of the country, which,
if left to private initiative, would distort the national objectives;

www.iasbaba.com

54

Economics- Subject wise compilation of 60 Day Plan- 2016


4. To achieve balanced regional development and dispersal of economic activity through
growth and diversification of economic activity in less developed areas by providing
adequate infra-structure and undertaking programmes of conservation and development of
national resources;
5. To reduce disparities in income;
6. To avoid concentration of economic powder in a few hands;
7. To exercise social control and regulation of long-term finance through public financial
institutions;
8. To control over sensitive areas, i.e., allocation of scarce imported commodities; control over
the distribution system in relation to essential goods in order to reduce the margin between
prices obtained by the producers and those paid by the consumers;
9. To attain self-reliance in different technologies through development of capacity for design
and development of machinery, equipment and instruments and elimination of dependence
of foreign agencies for these services;
10. To enhance the employment opportunities by heavy investment in industry and mining,
transport and communication; and
11. To increase exports and earn foreign exports and earn foreign exchange to ease the
pressure of Balance of Payments.
Q.103) Which of the following are correct w.r.t. Public Sector Undertakings (PSUs)?
1. The government-owned corporations are termed as Public Sector Undertakings (PSUs) in
India.
2. In a PSU majority (51% or more) of the paid up share capital is held by central government
or by any state government or partly by the central governments and partly by one or more
state governments.
3. The Comptroller and Auditor General of India (CAG) doesnt audit government companies.
4. In respect of government companies, CAG has the power to appoint the Auditor and to
direct the manner in which the Auditor shall audit the company's accounts.
Select the correct answer
a) 1 and 2 only
b) 1, 2 and 4only
c) 2, 3 and 4 only
d) All of the above
Q.103) Solution (b)
It is the CAG which audits the PSUs and the audit reports are verified by the Committee on
Public Sector Undertakings.
Q.104) Which of the following is incorrect about Public Enterprises Selection Board (PESB)?
a) The Public Enterprises Selection Board [P.E.S.B] is a high powered body constituted by
Government of India Resolution.

www.iasbaba.com

55

Economics- Subject wise compilation of 60 Day Plan- 2016


b) The P.E.S.B has been set up with the objective of evolving a sound managerial policy for the
Central Public Sector Enterprises and, in particular, to advise Government on appointments
to their top management posts.
c) To advise the Government on formulation and enforcement of a code of conduct and ethics
for managerial personnel applicable to PSEs and all government services.
d) To advise the Government on a suitable performance appraisal system for both the PSEs and
the managerial personnel in such enterprises
Q.104) Solution (c)
The code of conduct designed by PESB is applicable only to PSEs and not all services.
Further reading: http://pesb.gov.in/
Q.105) Consider the following about industrial policy 1991.
1. One of the major objectives was to liberalise the industry from regulatory devices such as
licenses and controls.
2. Ensuring PSUs are not exposed to competition and protected from private sector push.
Select the correct answer:
a) 1 only
b) 2 only
c) Both 1 and 2
d) None of the above
Q.105) Solution (a)
Introducing competition and ensuring PSUs are run on business lines with no undue protection
was the major objective of the industrial policy 1991.
Q.106) Consider the following about MRTP (Monopolies and Restrictive Trade Practices)Act.
1. The act came into force from 1st June, 1970 aiming to prevent concentration of economic
power, provide for control of monopolies, and protect consumer interest. Currently, the
MRTP Act has been repealed as the Competition Act, 2002, with a few changes to it.
2. A Monopolistic Trade Practice is that which represents abuse of market power in production
and marketing of goods and services by eliminating potential competitors, charging
unreasonably high prices, preventing or reducing competition, limiting technical
development, deteriorating product quality, etc.
Select the correct answer:
a) 1 only
b) 2 only
c) Both 1 and 2
d) None of the above
Q.106) Solution (c)

www.iasbaba.com

56

Economics- Subject wise compilation of 60 Day Plan- 2016


Self Explanatory
Further reading: http://www.slideshare.net/Dread_95/mrtpmonopolies-and-restrictive-tradepractices-act (content verified)

*Note: Statement (1) is modified renamed to repealed

Q.107) Which of the following is incorrect about Competition Commission of India (CCI)?
a) CCI is a statutory body established under the Competition Act 2002.
b) The Act prohibits anti-competitive agreements, abuse of dominant position by enterprises
and regulates combinations (acquisition, acquiring of control and M&A), which causes or
likely to cause an appreciable adverse effect on competition within India applicable only for
state and centre owned establishments.
c) The Commission is also required to give opinion on competition issues on a reference
received from a statutory authority established under any law and to undertake competition
advocacy, create public awareness and impart training on competition issues.
d) To promote and sustain an enabling competition culture through engagement and
enforcement that would inspire businesses to be fair, competitive and innovative; enhance
consumer welfare; and support economic growth.
Q.107) Solution (b)
CCI applicable across India not limiting to government owned or private; Competition is a larger
concept is well understood.
Q.108) Corporate Governance refers to the way a corporation is governed. Consider the
following.
1. Narayan Murthy Committee was set up by SEBI in 2003 w.r.t Corporate governance.
2. Whistle blower policy was a key recommendation of the committee.
3. The committee observed nominee of the Government on public sector companies shall be
elected and shall be subject to the same responsibilities and liabilities as other directors.
Select the correct answer
a) 1 and 2 only
b) 1 and 3 only
c) 2 and 3only
d) All of the above
Q.108) Solution (d)
A question of such nature can be not attempted until one has knowledge of the same. But the
concept of corporate governance is important for this year.
Further reading:

www.iasbaba.com

57

Economics- Subject wise compilation of 60 Day Plan- 2016


http://shodhganga.inflibnet.ac.in/bitstream/10603/8683/11/11_chapter%203.pdf
Q.109) Which of the following are Maharatnas?
1. Bharat Heavy Electricals Limited
2. Coal India Limited
3. GAIL (India) Limited
4. Indian Oil Corporation Limited
5. NHPC Limited
6. Oil & Natural Gas Corporation Limited
7. Steel Authority of India Limited
Select the correct answer.
a) 1, 2 ,3, 4, 6 and 7 only
b) 2 ,3, 4, 5, 6 and 7 only
c) 1, 2 ,3, 4, 5 and 7 only
d) All of the above
Q.109) Solution (a)
It is not NHPC but NTPC. (oh!!!! No!!!!)
Further reading: http://dpe.nic.in/publications/list_of_maharatna_navratna-and_miniratna
Q.110) The Department of Disinvestment has been renamed as Department of Investment
and Public Asset Management (DIPAM). Consider the following.
1. It is a department under the ministry of Heavy Industries and Public Enterprises.
2. The new department has been mandated to "advise the government in the matters of
financial restructuring of central public sector enterprises and for attracting investment
through capital markets".
3. The Cabinet Committee on Economic Affairs(CCEA), chaired by the Prime Minister gives its
approval for the procedure and mechanism for strategic disinvestment.
Select the correct answer
a) 1 and 2 only
b) 1 and 3 only
c) 2 and 3only
d) All of the above
Q.110) Solution (c)
It is a department under Ministry of Finance. Be careful with the ministry departments.
Further reading: http://pib.nic.in/newsite/PrintRelease.aspx?relid=137044
Q.111) Strategic sale is a part of disinvestment. Consider the following.
1. The NITI Aayog will identify the CPSE for strategic sale.

www.iasbaba.com

58

Economics- Subject wise compilation of 60 Day Plan- 2016


2. The previous NDA government had between 1999 and 2004 privatised about a dozen stateowned firms and hotels including Videsh Sanchar Nigam Ltd (VSNL), Bharat Aluminium
Company Ltd (BALCO), CMC Ltd and Hindustan Zinc (HZL).
Select the correct answer:
a) 1 only
b) 2 only
c) Both 1 and 2
d) None of the above
Q.111) Solution (c)
The above two statements are correct. In the strategic sale of a company, the transaction has
two elements:
Transfer of a block of shares to a Strategic Partner and
Transfer of management control to the Strategic Partner
Q.112) Consider the following regarding Central Pay Commission:
1. It is a statutory body formed after Pay Commissions Act 1947.
2. It is constituted in every five years.
3. It is set to review and make recommendations on the work and pay structure of all civil and
military divisions of the Government of India.
Which of the above statements are correct?
a) 1 and 2
b) 3 only
c) 2 and 3
d) 1 and 3
Q.112) Solution (b)
Pay Commission is set up intermittently by Government of India, and gives its recommendations
regarding changes in salary structure of its employees. Since India's Independence, seven pay
commissions have been set up on a regular basis to review and make recommendations on the
work and pay strcture of all civil and military divisions of the Government of India.
Headquartered in Delhi, the Commission is given 18 months from date of its constitution to
make its recommendations.
Q.113) Which of the following dimensions are included in calculating Social Progress Index
(SPI)?
1. Basic needs for survival
2. Access to the building blocks to improve living conditions
3. Access to opportunity to pursue goals and ambitions
Select the code from below:

www.iasbaba.com

59

Economics- Subject wise compilation of 60 Day Plan- 2016


a)
b)
c)
d)

1 and 2
2 and 3
1 and 3
All of the above

Q.113) Solution (d)


The Social Progress Index measures the extent to which countries provide for the social and
environmental needs of their citizens. Fifty-four indicators in the areas of basic human needs,
foundations of wellbeing, and opportunity to progress show the relative performance of
nations. The index is published by the nonprofit Social Progress Imperative, and is based on the
writings of Amartya Sen, Douglass North, and Joseph Stiglitz.[1] The SPI measures the well-being
of a society by observing social and environmental outcomes directly rather than the economic
factors. The social and environmental factors include wellness (including health, shelter and
sanitation), equality, inclusion, sustainability and personal freedom and safety.
The Index combines three dimensions
1. Basic Human Needs
2. Foundations of Well-being, and
3. Opportunity
Q.114) Consider the following statements regarding the Expenditure Management
Commission:
1. The commission was set up by the Government under the Chairmanship of Dr. Bimal Jalan.
2. It objective is to suggest ways of meeting development expenditure needs without
compromising on Fiscal Discipline.
Which of the above statements are correct?
a) 1 only
b) 2 only
c) Both 1 and 2
d) Neither 1 nor 2
Q.114) Solution (c)
The government in September 2014 decided to constitute an expenditure Management
Commission under the Chairmanship of Dr. Bimal Jalan.
The commission was asked to:
Review major areas of Central Government expenditure, the institutional arrangements
(Such as budgeting process)

Suggest ways of meeting development expenditure needs without compromising on


Fiscal Discipline.
Suggest a strategy to improve allocative efficiency, to meet reasonable proportion of
expenditure on services through user charges,etc.
Consider any other relevant issue concerning public expenditure management.
www.iasbaba.com

60

Economics- Subject wise compilation of 60 Day Plan- 2016


Recommendations of the report have not been made public yet.
Q.115) The Ministry of Micro, Small and Medium enterprises released a draft framework for
revival of MSMEs order,2015. Which of the following statements regarding the draft
framework are correct?
1. Banks are required to classify MSMEs into three categories, based on overdue principle or
interest payments.
2. MSMEs that predict a failure of business or inability to pay debts can voluntarily initiate
proceedings under the proposed Framework.
Select the code from the following:
a) 1 only
b) 2 only
c) Both 1 and 2
d) Neither 1 nor 2
Q.115) Solution (c)
The draft framework seeks to enable sick MSMEs to obtain financial support.
Key provisions for draft framework include:
Classification of Sick MSMEs
Committees for distressed MSMEs
Corrective Action plan
Do read it in detail.
Q.116) The action plan for the Start Up India initiative was released by the Prime Minister, Mr.
Narendra Modi in Jan 2016. The initiatives seeks to encourage start-ups in India, with the aim
of promoting sustainable economic growth and generating large scale employment. Which of
the following statements are correct about Start Up India initiative?
1. Start Ups will be allowed to self certify their compliance with six labour laws.
2. In case of Labour Laws and environmental Laws, inspections will not be conducted for three
years, unless a complaint is received.
Select the code from below:
a) 1 only
b) 2 only
c) Both 1 and 2
d) Neither 1 nor 2
Q.116) Solution (a)
In case of Labour Laws inspections will not be conducted for three years, unless a complaint
is received.
For Environmental laws, random checks would be carried out to ensure compliance.

www.iasbaba.com

61

Economics- Subject wise compilation of 60 Day Plan- 2016


Q.117) Consider the following statements with regard to index of Industrial Production (IIP)
1) IIP conveys the status of production in the industrial sector of an economy in a given period
of time, in comparison with a fixed reference point in the past.
2) In the IIP index, the maximum weightage is given to Basic goods, followed by Consumer
Goods, followed by Intermediate and Capital Goods.
Select the correct code:
a) Only 1 is correct
b) Only 2 is correct
c) Both 1 and 2 is correct
d) Neither 1 nor 2 is correct
Q.117) Solution (c)
Explanation:
Index of Industrial Production (IIP) is an abstract number, the magnitude of which
represents the status of production in the industrial sector for a given period of time as
compared to a reference period of time It is a statistical device which enables us to arrive at
a single representative figure to measure the general level of industrial activity in the
economy.
Strictly speaking the IIP is a short term indicator measuring industrial growth till the actual
result of detailed industrial surveys become available. This indicator is of paramount
importance and is being used by various organisations including Ministries/Departments of
Government of India, Industrial Associations, Research Institutes and Academicians.
In India, IIP is a representative figure which measures the general level of Industrial activity
in the country. Being an abstract number, it does not show volume of activity and only
shows the magnitude which represents the status of production in the industrial sector for a
given period of time as compared to a reference period of time.
The maximum weightage is given to Basic goods, followed by Consumer Goods, followed by
Intermediate and Capital Goods. Among Consumer goods, the weightage of consumer nondurables is much more than of durables.

www.iasbaba.com

62

Economics- Subject wise compilation of 60 Day Plan- 2016

Q.118) Which of the following statements is true about 'P-Notes' in India?


1) These are issued by SEBI in India.
2) They cannot be used within the country.
3) Investors in P-Notes are not required to register with SEBI.
4) Due to the issue of money laundering, P-Notes are banned in India.
Select the correct answer using the code given below.
a) 1, 2 and 3 only
b) 2 and 3 only
c) 1 and 4 only
d) 1, 2, 3 and 4
Q.118) Solution (b)
Explanation:
Participatory Notes commonly known as P-Notes or PNs are instruments issued by
registered foreign institutional investors (FII) to overseas investors, who wish to invest in the
Indian stock markets without registering themselves with the market regulator, the
Securities and Exchange Board of India - SEBI.
SEBI permitted foreign institutional investors to register and participate in the Indian stock
market in 1992. PNs have decreased but not completely banned.
A Participatory Note (PN or P-Note) in the Indian context, in essence, is a derivative
instrument issued in foreign jurisdictions, by SEBI and a registered Foreign Institutional
Investor (FII) or its sub-accounts or one of its associates, against underlying Indian securities.
PNs are also known as Overseas Derivative Instruments, Equity Linked Notes, Capped Return
Notes, and Participating Return Notes etc.
For more: http://www.business-standard.com/article/markets/5-things-to-know-about-pnotes-115072800122_1.html
Q.119) Consider the following statements regarding Universal Service Obligation Fund (USOF):
1) It is a statutory fund.
2) It is used to extend basic telephony services to the rural areas.

www.iasbaba.com

63

Economics- Subject wise compilation of 60 Day Plan- 2016


3) It is maintained as a part of Consolidated Fund of India.
4) Resources for USOF are raised exclusively through Universal Service Levy on revenues of all
Telecom Service Providers.
Which of the statements given above is/are correct?
a) 1 and 2 only
b) 2, 3 and 4 only
c) 2 and 4 only
d) 1, 2, 3 and 4
Q.119) Solution (a)
Explanation:
USOF is a statutory fund under Indian Telegraph Act, 2003. The objective is to extend basic
telecom services to the rural areas. For this purpose National Optical Fiber Network (NOFN)
is being funded by USOF. Hence, 1st and 2nd statements are correct.
USOF is maintained outside the Consolidated Fund, and is maintained by the Department of
Telecommunication, Ministry of Communication. Hence, 3rd statement is incorrect.
Apart from the Levy the resources are provided by the Central Government in the form of
grants and loans. Hence, 4th statement is incorrect.
Q.120) Recently, there has been large contraction in Indias trade surplus in services. It is due
to:
a) sharp drop in non-software services exports
b) global economic slowdown
c) drop in software services exports
d) All of the above
Q.120) Solution (d)
Explanation:
Indias trade surplus in services has been contracting, mainly due to a sharp drop in nonsoftware services exports
According to economists, the global economic slowdown is finally beginning to affect Indias
services sector.
Both software services and non-software services exports are on decline due to global
economic slowdown.
Link: Slowdown hits services sector http://www.thehindu.com/business/Industry/slowdownhits-services-sector/article8505380.ece
Q.121) Identify the organization that prepared a standardised electronic format for
information exchange on Base Erosion and Profit Sharing (BEPS) to help tax authorities
a) Organisation for Economic Cooperation and Development (OECD)
b) Agency for Cooperation and Research in Development (ACORD)
www.iasbaba.com

64

Economics- Subject wise compilation of 60 Day Plan- 2016


c) International Bank for Reconstruction and Development (IBRD)
d) International Monetary Fund (IMF)
Q.121) Solution (a)
Explanation:
There have been concerns across the globe about companies making profits in a particular
country but not paying taxes to the local government. The Organization for Economic
Cooperation and Development (OECD) states that BEPS is of major significance for
developing countries due to their heavy reliance on corporate income tax, particularly from
multinational enterprises. It also states that estimates since 2013 conservatively indicate
annual losses of anywhere from 4 to10 per cent of global corporate income tax revenues, or
$100-$240 billion annually.
The OECD, under the authority of the Group of 20 countries, has considered ways to revise
tax treaties, tighten rules, and to share more government tax information under the BEPS
project, and has issued action plans last year. One of the areas discussed was on addressing
tax challenges in the digital economy.
Link:
Whats
Base
Erosion
and
Profit
http://www.thehindu.com/news/cities/mumbai/whats-base-erosion-and-profitshifting/article8404293.ece

Shifting?

Q.122) Consider the following about Current Account Deficit.


1. A measurement of a countrys trade in which the value of goods and services it imports
exceeds the value of goods and services it exports.
2. If a current account deficit is financed through borrowing it is said to be more sustainable
and is the globally accepted method.
3. A factor behind the Asian crisis of 1997 was that countries had run up large current account
deficits by attracting capital flows (hot money) to finance the deficit.
4. A current account deficit, may imply that you are relying on consumer spending, and are
becoming uncompetitive.
Select the correct answer
a) 1 and 2 only
b) 2 and 3 only
c) 1, 3 and 4 only
d) All of the above
Q.122) Solution (c)
If a current account deficit is financed through borrowing it is said to be more unsustainable.
This is because borrowing is unsustainable in the long term and countries will be burdened with
high interest payments. E.g Russia was unable to pay its foreign debt back in 1998. Other
developing countries have experience similar repayment problems Brazil, African countries (3rd
World debt) Countries with large interest payments have little left over to spend on investment.

www.iasbaba.com

65

Economics- Subject wise compilation of 60 Day Plan- 2016

Q.123) consider the following about J-curve.


1. A theory stating that a country's trade deficit will worsen initially after the depreciation of its
currency because higher prices on foreign imports will be greater than the reduced volume
of imports.
2. In private equity funds , the J-curve effect occurs when funds experience negative returns
for the first several years.
Select the correct answer:
a) 1 only
b) 2 only
c) Both 1 and 2
d) None of the above
Q.123) Solution (c)
Both the statements are correct. It is true for equity funds too.
An example of the J-curve effect is seen in economics when a country's trade balance initially
worsens following a devaluation or depreciation of its currency. The higher exchange rate will at
first correspond to more costly imports and less valuable exports, leading to a bigger initial
deficit or a smaller surplus. Due to the competitive, relatively low-priced exports, however, a
country's exports will start to increase. Local consumers will also purchase less of the more
expensive imports and focus on local goods. The trade balance eventually improves to better
levels compared to before devaluation.
Q.124) The balance of payments (BOP) is the method countries use to monitor all
international monetary transactions at a specific period of time. Consider the following.
1. All trades conducted by both the private and public sectors are accounted for in the BOP in
order to determine how much money is going in and out of a country.
2. If a country has received money, this is known as a debit, and if a country has paid or given
money, the transaction is counted as a credit.
3. Theoretically, the BOP should be zero, meaning that assets (credits) and liabilities (debits)
should balance, but in practice this is rarely the case.
Select the correct answer
a) 1 and 2 only
b) 1 and 3 only
c) 2 and 3only
d) All of the above
Q.124) Solution (b)
We all understand what is credit and debit. Just a reading mistake can cause an error here. So
beware!!!

www.iasbaba.com

66

Economics- Subject wise compilation of 60 Day Plan- 2016

Q.125) Which of following is not under current account of balance of payments (BOP)?
a) Goods
b) Services
c) Loans
d) Transfers
Q.125) Solution (c)
Goods, Services, Income and Transfers are in Current account and Investment, Loan and Banking
Capital are in Capital Account.

Q.125) Consider the following. Identify the incorrect statement.


a) India experiences a deficit in income part of current account in BOP.
b) India experiences a surplus in services part of current account in BOP.
c) India experiences a deficit in transfers part of current account in BOP.
d) All the above are correct
Q.126) Solution (c)
Transfersis nothing but remittances and India has the highest remittances in the world
followed by China. Hence Surplus
*Note: In question, it was missed to mention Identify the incorrect statement.
Q.127) Which of the following will lead to reduction of CAD?
1. Decrease in crude oil consumption.
2. Increase in import duty of gold.
3. Boost to IT sector in India.
4. Promote FDI
Select the correct answer
a) 1, 2 and 3 only
b) 1, 2 and 4 only
c) 1, 3 and 4 only
d) All of the above
Q.127) Solution (d)
It is self explanatory. Boost to It sector will increase exports and hence.
Q.128) Consider the following about Arvind Mayaram Committee w.r.t foreign investments.
1. Foreign investment of 10 per cent or more in a listed company will now be treated as
foreign direct investment (FDI).

www.iasbaba.com

67

Economics- Subject wise compilation of 60 Day Plan- 2016


2. It said an investor may be allowed to invest below 10 per cent and this can be treated as
FDI subject to the condition that the FDI stake is raised to 10 per cent or beyond within one
year from the date of the first purchase.
3. The panel has suggested that foreign investment in an unlisted company, irrespective of the
threshold limit, may be treated as FPI.
4. Any investment by way of equity shares, compulsorily convertible preference
shares/debentures less than 10 per cent should treated as Foreign Portfolio Investment
(FPI).
Select the correct answer
a) 1 and 3 only
b) 1, 2 and 4 only
c) 2, 3 and 4 only
d) 1, 2 and 3 only
Q.128) Solution (b)
It is not outdated if we are thinking so. The recommendations are valid even today.
The panel has suggested that foreign investment in an unlisted company, irrespective of the
threshold limit, may be treated as FDI not FPI.
Further reading:
http://finmin.nic.in/the_ministry/dept_eco_affairs/investment_division/Report%20of%20Dr%2
0Arvind%20Mayaram%20committe%20on%20FDI_FII.pdf
Q.129) Consider the following.
1. RBI and Ministry of Finance monitors the FDI investment.
2. SEBI is the regulator for FPI, FII or RFPI investments.
Select the correct answer:
a) 1 only
b) 2 only
c) Both 1 and 2
d) None of the above
Q.129) Solution (c)
The regulators for both kind of investments are important to note. Both the above are correct.
This is sector specific regulation but inter department cooperation will always be there.
Q.130) Consider the following.
1. If BOP is positive then it might lead to rupee appreciation and hence good for the exporters.
2. If BOP is negative then it might lead to rupee depreciation and hence the RBI will absorb the
excess rupee in the market.
Select the correct answer:

www.iasbaba.com

68

Economics- Subject wise compilation of 60 Day Plan- 2016


a)
b)
c)
d)

1 only
2 only
Both 1 and 2
None of the above

Q.130) Solution (b)


Rupee appreciation is always bad for the exporters. Similarly when BOP is negative it will lead to
rupee depreciation and hence excess rupee in the market. RBI will attempt to absorb to
maintain balance. Hence a is wrong and b is right.
Q.131) Consider the following about nominal and real exchange rate.
1. Nominal exchange rate is the price of one currency in terms of number of units of some
other currency.
2. It is 'nominal' because it measures only the numerical exchange value, and does not say
anything about other aspects such as the purchasing power of that currency.
3. The real exchange rates are nothing but the nominal exchange rates multiplied by the price
indices of the two countries.
4. The real exchange rates are nothing but the nominal exchange rates multiplied by the price
indices of the two countries.
Select the correct answer
a) 1, 2 and 3 only
b) 1, 2 and 4 only
c) 1, 3 and 4 only
d) All of the above
Q.131) Solution (d)
Pure and simple understanding of the concepts is put in words. This will help better
comprehension if any.
Further reading:
http://articles.economictimes.indiatimes.com/2003-01-27/news/27555718_1_real-exchangereer-nominal-effective-exchange-rate
Q.132) Consider the following statements regarding the Exchange Rate Systems:
1. Under a freely floating exchange rate regime, authorities do not intervene in the market for
foreign exchange and there is minimal need for international reserves.
2. RBI does not intervene in Foreign Exchange market, with the objective of maintaining the
Real Effective Exchange Rate stable.
Which of the above statements are correct?
a) 1 only
b) 2 only
c) Both 1 and 2

www.iasbaba.com

69

Economics- Subject wise compilation of 60 Day Plan- 2016


d) Neither 1 nor 2
Q.132) Solution (a)
The current exchange rate regime, introduced in 1993, the RBI has been, actively intervening in
the FOREX market with the objective of maintaining the Real Effective Exchange Rate (REER)
stable.
Q.133) Consider the following statements:
The price of any currency in international market is decided by the
1. World Bank
2. Demand for goods/services provided by the country concerned
3. Stability of the government of the concerned country
4. Economic potential of the country in question
Which of the statements given above are correct?
a) All of the above
b) 1 and 4
c) 2 and 3
d) 3 and 4
Q.133) Solution (c)
Previous year question from UPSC. Direct question.
Q.134) The value of the SDR is currently based on a basket of four major currencies: the U.S.
dollar, euro, the Japanese yen, and pound sterling. The basket is being expanded to include
which of the following currencies?
a) Indian Rupee
b) Swiss Franc
c) Saudi Riyal
d) Chinese Renminbi
Q.134) Solution (d)
The SDR is an international reserve asset, created by the IMF in 1969 to supplement its member
countries official reserves. As of March 2016, 204.1 billion SDRs (equivalent to about $285
billion) had been created and allocated to members. SDRs can be exchanged for freely usable
currencies. The value of the SDR is currently based on a basket of four major currencies: the U.S.
dollar, euro, the Japanese yen, and pound sterling. The basket will be expanded to include the
Chinese renminbi (RMB) as the fifth currency, effective October 1, 2016.
Q.135) If the foreign-currency equivalent of the home currency falls, then
1. the home currency has devalued.
2. the home currency has revalued.
3. the currency must be in a freely fluctuating exchange-rate system.

www.iasbaba.com

70

Economics- Subject wise compilation of 60 Day Plan- 2016


4. the currency is less valuable in purchasing power parity.
Which of the above statements are incorrect?
a) 1 only
b) 2 only
c) 3 and 4
d) 2,3 and 4
Q.135) Solution (d)
If the foreign currency equivalent of the home currency falls, then the home currency has
devalued.
The currency may or may not be in freely fluctuating exchange rate system.
Purchasing Power Parity is not directly related with market exchange rate of currencies.
Q.136) Which of the following are the functions of IMF?
1. provide emergency loans to countries facing balance of payments problems.
2. monitor macroeconomic developments continuously in member countries.
3. serve as the world central bank.
4. to provide a line of credit for each member country.
Select the code from following :
a) 1 and 2
b) 2 and 3
c) 1,2 and 4
d) All of the above
Q.136) Solution (c)
IMF does not serve as World Central Bank.
Q.137) Consider the following statements with regard to Central Statistics Office (CSO) under
Ministry of Statistics and Programme Implementation:
1) The CSO releases Consumer Price Indices (CPI) for all-India and states/union territories
separately for rural, urban, industrial workers and combined (rural plus urban)
2) CSO also disseminates Energy Statistics, Social and Environment Statistics and prepares the
National Industrial Classification
Which of the statements given above is/are correct?
a) 1 only
b) 2 only
c) Both 1 and 2
d) Neither 1 nor 2
Q.137) Solution (b)

www.iasbaba.com

71

Economics- Subject wise compilation of 60 Day Plan- 2016


Explanation:
Consumer Price Indices (CPI) for Industrial Workers are released by Labour Bureau,
Government of India.
The Central Statistics Office (CSO), an attached office of the ministry, coordinates the
statistical activities in the country and evolves statistical standards.
Its activities inter-alia, include compilation of National Accounts, Index of Industrial
Production, Consumer Price Indices for Urban/Rural/ Combined, Human Development
Statistics, including Gender Statistics in the states and union territories and disseminates
Energy Statistics, Social and Environment Statistics and prepares the National Industrial
Classification.
Q.138) Which among the following statements is/are correct in regard to the Cash based
Accounting System and Accrual Accounting System?
1) The Indian Government accounts are prepared on a cash based accounting system, which
does not give a realistic account of governments financial position.
2) The present system does not reflect accrued liabilities arising from the gap between
communities and transactions of government on the one hand and payments made.
3) The 12th Finance Commission recommended for the introduction of accrual accounting in
Government, which have been accepted in principle.
Select the correct one/ones, using the code given below:
a) 1 and 2
b) 2 and 3
c) 1 and 3
d) 1, 2 and 3
Q.138) Solution (d)
Explanation:
The Indian Government accounts are prepared on a cash based accounting system. This
system recognizes a transaction when cash is paid or received. However, it does not give a
realistic account of government's financial position because it lacks an adequate framework
for accounting for assets and liabilities, and depicting consumption of resources.
Moreover, capital expenditure (expenditure on the creation of new assets) under the cash
system is brought to account only in the year in which a purchase or disposal of an asset is
made. This is not an effective way to track assets created out of public money.
The present system does not reflect accrued liabilities arising from the gap between
commitments and transactions of government on the one hand and payments made.
The Twelfth Finance Commission recommended introduction of accrual accounting in
Government. Government has accepted the recommendation in principle and asked
Government Accounting Standards Advisory Board (GASAB) in the office of the Comptroller
and Auditor General of India (CAG) t draw a roadmap for transition from cash to accrual
accounting system and to prepare an operational framework for its implementation.

www.iasbaba.com

72

Economics- Subject wise compilation of 60 Day Plan- 2016

So far twenty one State Governments have agreed in principle to introduce accrual
accounting.

Q.139) Select the incorrect one/ones about the Most Favoured Nation (MFN) provisions of the
WTO from the given list of statements, using the code given below:
1) In general, MFN means that every time a country lowers a trade barrier or opens up a
market, it has to do so for the same goods or services for all its trading partners
2) The MFN clause functions differently for the developed and developing countries
Select the appropriate code
a) 1 only
b) 2 only
c) Both 1 and 2
d) Neither 1 nor 2
Q.139) Solution (b)
Explanation:
The MFN provisions of the WTO functions the same way for all its trading partners whether
they are developed or developing countries.
Q.140) Consider the following statements in regard to the concept known as Labour force
participation rate in India:
1) The Labour Force Participation Rate (LFPR), obtained by dividing the number of persons in
the labour force by total population, is an important parameter in employment projections
and formulation of employment strategies.
2) The National Sample Survey Organisation (NSSO) defines labour force participation rate
(LFPR) as the number of persons/person days in labour force per thousand persons/person
days.
Select the correct answer using the code given below
a) Only 1
b) Only 2
c) 1 and 2
d) Neither 1 nor 2
Q.140) Solution (c)
Explanation:
Both of the statements are correct about the concept of the labour force participation ratio
(LFPR) in India.
To calculate the LFPR at first the number or persons/person days employed and the number
of persons/person days unemployed are added--addition is multiplied by 1000 and the sum
is divided by the total population.

www.iasbaba.com

73

Economics- Subject wise compilation of 60 Day Plan- 2016


Doubts: Baba ji... in 24th question..LFPR.. isn't the denominator "eligible population"..?
Then what is Work Participation Rate... Kindly help to clarify my doubt..thank you..
Clarification: Refer this link (Go to - Chapter Three Key Findings of NSS 66th round (July
2009- June 2010) survey on Employment and Unemployment Pg.8 Last para) or Cntrl+F
and search Labour force participation rate
http://mospi.nic.in/Mospi_New/upload/Key_Indicators_Emp_&_Unemp_66th_round.pdf
or
http://www.arthapedia.in/index.php?title=Labour_Force_Participation_Rate

It has given - Labour-force participation rate (LFPR) is defined as the proportion of


persons/person days in the labour-force to the total persons/person-days.
Worker Population Ratio (WPR): WPR defined as the number of persons/person days
employed per 1000 persons/person-days.
Q.141) Consider the following statements in regard to agricultural schemes in India:
1) National Food Security Mission (NFSM) to increase the production of rice, wheat and pulses
by 10,8 and 2 million tonnes, respectively by the end of the 12th Plan.
2) The Government of India has approved crop development programme on cotton, jute and
sugarcane for enhancing productivity under National Food Security Mission-Commercial
Crops (NFSM-CC) from 2014-15.
3) Krishonnati Yojana is a new umbrella scheme and the above two schemes have been
included in Krishonnati scheme.
Which of the statements given above is/are correct?
a) 2 and 3 only

www.iasbaba.com

74

Economics- Subject wise compilation of 60 Day Plan- 2016


b) 1 and 2 only
c) 1 and 3 only
d) All of the above
Q.141) Solution (a)
Explanation:
National Food Security Mission (NFSM) to increase the production of rice, wheat, pulses and
coarse cereals by 10,8,4 and 3 million tonnes, respectively by the end of the 12th Plan.
Q.142) Which of the following are part of World bank group?
1. The International Bank for Reconstruction and Development
2. The International Development Association
3. The International Finance Corporation
4. The Department of International Development
5. The Multilateral Investment Guarantee Agency
6. The World Intellectual Property Organisation
7. The International Centre for Settlement of Investment Disputes
Select the correct answer.
a) 1, 3, 4, 5 and 7 only
b) 1, 2 ,3, 5 and 7 only
c) 1, 2, 4, 5 and 7 only
d) All of the above
Q.142) Solution (b)
The Department for International Development (DFID) leads the UKs work to end extreme
poverty, building a safer, healthier, more prosperous world for all of us which is firmly in the
UKs national interest.
WIPO is the global forum for intellectual property services, policy, information and cooperation.
It is a UN organisation.
Q.143) Consider the following about World Intellectual Property Organisation (WIPO).
1. World Intellectual Property Organisation is a member of the World Bank group.
2. WIPO was created in 1967 "to encourage creative activity, to promote the protection of
intellectual property throughout the world.
3. The predecessor to WIPO was the BIRPI (Bureaux Internationaux Runis pour la Protection
de la Proprit Intellectuelle, French acronym for United International Bureaux for the
Protection of Intellectual Property)
4. WIPO's member states and stakeholders share a common goal: an efficient and accessible
intellectual property system that provides benefits to all.
Select the correct answer
a) 1 and 2 only

www.iasbaba.com

75

Economics- Subject wise compilation of 60 Day Plan- 2016


b) 1, 2 and 4only
c) 2, 3 and 4 only
d) All of the above
Q.143) Solution (c)
The World Intellectual Property Organization (WIPO) is one of the 17 specialized agencies of the
United Nations. WIPO was created in 1967 "to encourage creative activity, to promote the
protection of intellectual property throughout the world. It is not a World Bank group member.
Q.144) Consider the following.
1. Protectionism refers to government actions and policies that restrict or restrain
international trade, often done with the intent of protecting local businesses and jobs from
foreign competition.
2. Import substitution industrialization (ISI) is a theory of economics typically utilized by
developing countries or emerging market nations seeking to decrease dependence on
developed countries and to increase self-sufficiency.
Select the correct answer:
a) 1 only
b) 2 only
c) Both 1 and 2
d) None of the above
Q.144) Solution (c)
Both are correct and copybook definitions.
Q.145) Which of the following is not correctly matched w.r.t. modes of supply?
a) Mode 1 Cross Border Supply
b) Mode 2 service delivered within the territory of the member through presence of the
supplier
c) Mode 3 Commercial presence
d) Mode 4 Presence of the natural person
Q.145) Solution (b)
Mode 2 is Consumption abroad and the service provider is not present within the territory of
the member.
Q.146) Consider the following about WTO.
1. With GATS the multilateral trading system includes services for the first time as its
predecessor GATT didnt include services.
2. The Anti Counterfeiting Trading Agreement is under the arrangement of WTO and
establishes international standards w.r.t. IPRs.

www.iasbaba.com

76

Economics- Subject wise compilation of 60 Day Plan- 2016


Select the correct answer:
a) 1 only
b) 2 only
c) Both 1 and 2
d) None of the above
Q.146) Solution (a)
The ACTA is out of WTO and WIPO and a multinational treaty establishing international
standards w.r.t IPRs. It is by the countries like Japan, USA, Australia, New Zealand, Canada etc.
Q.147) Which of the following states were recently granted GI for Basmati rice?
1. Jammu and Kashmir
2. Madhya Pradesh
3. Haryana
4. Western Uttar Pradesh
5. Punjab
6. Himachal Pradesh
Select the correct answer.
a) 1, 2 ,3, 4 and 6 only
b) 2 ,3, 4, 5 and 6 only
c) 1, 3, 4, 5 and 6 only
d) 1, 2, 3, 5 and 6 only
Q.147) Solution (c)
Madhya Pradesh lost its bid.
Further reading:
http://www.thehindu.com/news/national/indian-basmati-rice-gets-gi-tag/article8198492.ece
Q.148) Consider the following.
1. The Asian Development Bank (ADB) is an Asia regional development organization dedicated
to reducing poverty in Asia and the Pacific through loans, grants with headquarters in
Beijing.
2. The Asian Infrastructure Investment Bank (AIIB) is an international financial institution that
aims to support the building of infrastructure in the Asia-Pacific region with headquarters in
Manila, Philippines.
3. The New Development Bank (NDB), formerly referred to as the BRICS Development Bank, is
a multilateral development bank established by the BRICS states with head quarters in
Shanghai, China.
Select the incorrect answer
a) 1 and 2 only
b) 1 and 3 only

www.iasbaba.com

77

Economics- Subject wise compilation of 60 Day Plan- 2016


c) 2 and 3only
d) All of the above
Q.148) Solution (a)
Headquarters have been mingled around. These are important institutions keeping the changing
dynamics in mind.
Q.149 Which of the following is correct?
a) The UN Monetary and Financial Conference held post Second World War established the
international trading regime called General Agreement on Trade and Tariffs.
b) WTO is part of whole set up of UNO and thus is guided by the actions its member bodies.
c) IMF follow voting by consensus pattern by which each member gets to ensure their idea is
considered.
d) WTO doesnt have weighted voting as the World Bank or the IMF and goes by the system of
one member one vote.
Q.149) Solution (d)
The conference referred to in statement (a) is Bretton Wodds Conference and it lead to World
Bank and IMF coming to place.
WTO is not part of UN set up and is independent. However both share an agreement between
each other.
IMF follow weighted voting and the members with higher quota get a higher voice.
Q.150) Which of the following are correct w.r.t. Trans Pacific Partnership?
1. The Trans-Pacific Partnership (TPP) is a trade agreement among twelve Pacific Rim
countries.
2. Historically, the TPP is an expansion of the Trans-Pacific Strategic Economic Partnership
Agreement (TPSEP or P4) signed by Brunei, Chile, New Zealand, and Singapore in 2005.
3. The TPP contains measures to raise trade barriers, such as tariffs to rogue nations and
establish an investor-state dispute settlement mechanism (but states can opt out from
tobacco-related measures).
4. South Korea is not part of the TPP deal.
Select the correct answer
a) 1, 2 and 3 only
b) 1, 2 and 4 only
c) 1, 3 and 4 only
d) All of the above
Q.150) Solution (b)
There is no such provision called rogue nations in the deal and it basically looks to reduce the
trade barriers between member nations.

www.iasbaba.com

78

Economics- Subject wise compilation of 60 Day Plan- 2016


Q.151) Which of the following is not correctly matched?
a) TATP Europe and America
b) RCEP ASEAN
c) FTTA APEC
d) NAFTA Eurasian Union
Q.151) Solution (d)
NAFTA is the North American Free Trade Agreement is an agreement signed by Canada, Mexico,
and the United States, creating a trilateral trade bloc in North America.
FTTA is Free Trade Area of Asia Pacific championed by APEC.
Q.152) Consider the following factors:
1. Declining prices of a number of commodities
2. Turbulent financial markets
3. Volatile exchange rates
4. Horizontal distribution of divisible pool among the States
Choose the correct option which signifies being a factor towards the disturbed economic
landscape of the world:
a) 1, 2 and 4
b) 1, 2 and 3
c) 1 and 2 only
d) 3 and 4 only
Q.152) Solution(b)
Declining prices of a number of commodities (Crude Oil Prices Decline in Imports)
Turbulent financial markets (Equity Market)
Volatile exchange rates
Rupee has depreciated vis--vis the US dollar
Chinas growth and currency developments this year deteriorated
Appreciated against some
Exhibits extreme risk-aversion behaviour of global investors
commodities exporting
economies are thus, put under considerable stress
Let us talk about what Economic Survey tells us about the State of Indian Economy
Growth has been positive on the strength of domestic absorption, and the country has
registered a robust and steady pace of economic growth in 2015-16 as it did in 2014-15
Exhibited signs of improvement
Inflation, Fiscal deficit and Current Account Balance
Healthy growth rate of PCI (defined as Net National Income (NNI) divided by the estimated
population)
Progress in its contribution to the global growth of Gross Domestic Product (GDP) in
Purchasing Power Parity (PPP) terms

www.iasbaba.com

79

Economics- Subject wise compilation of 60 Day Plan- 2016


Growth in Gross Value Added (GVA) (reflects the supply or production side)
Prices:
Substantial decline in price of the Indian basket of crude oil contributed to the decline in
general inflation
Prices of essential commodities under check astute policies and management of inflation
by the government through buffer stocking, timely release of cereals and import of pulses
and moderate increase in Minimum Support Prices (MSP) of agricultural commodities
Decline in the inflation of housing (rent), transport, communication, education and other
services.
Banking
RBI:
Reduced the repo rate (from 7.75 per cent to 6.75 per cent in September, 2015)
Kept the policy repo rate unchanged
Sluggish growth of bank credit:
Incomplete transmission of the monetary policy as banks have not passed on the entire
benefit to borrowers
Unwillingness of the banks to lend credit on account of rising Non-performing Assets (NPA)
Worsening of corporate balance sheets, forcing them to put their investment decisions on
hold
Interest rates in the bond market being more attractive to borrowers
Financial Inclusion:
Increase in the opening of basic savings bank deposit accounts under the Pradhan Mantri
Jan Dhan Yojana
Launch of Universal social security system for all Indians, especially for the poor and the
underprivileged, in the insurance and pension sectors;
a) Pradhan Mantri Suraksha BimaYojana
b) Pradhan Mantri Jeevan Jyoti Bima Yojana
c) Atal Pension Yojana
(Make a note of the schemes and basic information regarding them)
Pradhan Mantri Mudra Yojana Micro Units Development Refinance Agency (MUDRA) Bank to
refinance last mile financers
a) Refinance products with a loan requirement up to Rs. 10 lakh
b) Support to micro-finance institutions by way of refinance
Gold In order to mobilize gold for productive purpose and to reduce the countrys reliance on
imports of gold, two main schemes were launched a) Sovereign Gold Bond Scheme
b) Gold Monetization Scheme
(Make a note of the schemes and basic information regarding them)
Growth in Industry:

www.iasbaba.com

80

Economics- Subject wise compilation of 60 Day Plan- 2016


Owing to Manufacturing: Improved manufacturing activity
Aided by robust growth in petroleum refining, automobiles, wearing apparels, chemicals,
electrical machinery and wood products and furniture
Provides the demand base for the products of many other growing sectors of the economy,
thereby creating substantial backward linkages.
Witnessing a deceleration in growth: Electricity, gas, water supply and related utilities, mining
& quarrying and construction activities
Overall BoP Health Comfortable owing to the following factors:
a) Lower trade deficit and modest growth in invisibles resulted in lower Current Account Deficit
(CAD),
b) Continued increase in Foreign Direct Investment (FDI) inflows and Non-resident Indian (NRI)
deposits
c) Net outflow of portfolio investment
Negative Territory:
Wholesale price inflation & Decline in consumer prices inflation
Agriculture Sector: lower than the average of last decade, mainly on account of it being the
second successive year of lower than- normal monsoon rains
Farm Sectorlow growth on account of two consecutive years of deficient south-west
monsoon (June-September) rainfall & Below normal post monsoon (October-December)
rains
Decline in the production of food-grains and oil- seeds
Indias exports are suffering: Weak growth in advanced and emerging economies and low
global commodity prices, particularly petroleum.
Note:
PPP: Represents the number of units of a country's currency required to purchase the same
amount of goods and services in the domestic market as the US dollar would purchase in the
United States, thus adjusting for purchasing power differentials between currencies in relevant
markets
Eight core sectors
Coal, crude oil, natural gas, refinery products, fertilizers, steel, cement and electricity
Have a total weight of nearly 38 per cent in the IIP
(The chapter State of the Economy (Economic Survey) has been covered above roughly Please
read it and revise it again. Additionally, go through all the schemes mentioned above)
Q.153) To which factor can the recent growth-revival of India be attributed to?
a) Fixed Capital Formation
b) Net Exports
c) Net Imports
d) Private Consumption
Q.153) Solution (d)
www.iasbaba.com

81

Economics- Subject wise compilation of 60 Day Plan- 2016

Signifies the consumption-led growth revival


Order: Food > Housing > Transport > Misc. goods & services > Cloth > Health > Education

Q.154) What does Gross fixed capital formation refer to?


a) Difference between exports and imports of goods and non-factor services
b) Difference between private and public expenditure
c) Value of new machinery and equipment plus the value of new construction activity
undertaken during the year
d) Both (a) and (b)
Q.154) Solution (c)
Owing to the growth in capital goods
Gross fixed capital formation (GCFs) expenditure component of GDP is second to Private
Consumption
Q.155) The Baltic Dry Index measures
a) The expenditure component of various sectors to the GDP
b) A freight index and an indicator of shipping services
c) The performance and contribution of the core industries in the Infrastructure of the country
d) The difference between Foreign Tourist Arrivals and Foreign Exchange Earnings by the
country
Q.155) Solution (b)
Baltic Dry Index
A freight index and an indicator of shipping services
Low: slowdown in Indias and the worlds merchandise trade as well as of overseas shipping
services
Q.156) Consider the following statements:
1. The Labour Force Participation Rate (LFPR) of women is higher than that of males in rural
areas but lower in urban areas.
2. The Worker Population Ratio (WPR) reflects exactly opposite pattern w.r.t the pattern
exhibited by the LFPR.
Choose the correct option:
a) Only 1
b) Only 2
c) Both 1 and 2
d) None of the above
Q.156) Solution (d)
The Labour Force Participation Rate (LFPR) of women is significantly lower than that of
males in both rural and urban areas.
www.iasbaba.com

82

Economics- Subject wise compilation of 60 Day Plan- 2016

The Worker Population Ratio (WPR) reflects similar patterns w.r.t that exhibited by LFPR.

Q.157) Which of the following taxes come under Direct Tax Collections?
1) Personal Income Tax
2) Corporate Tax
3) Security Transaction Tax
4) Wealth Tax
Select the correct code:
a) All of the above
b) 1, 2 and 3 only
c) 1, 2 and 4 only
d) 1 and 2 only
Q.157) Solution (a)

Q.158) Consider the following statements with regard to India and IMF:
1) India is the founder member of the IMF and also among the top 10 members of IMF
2) India was among the first five nations having the highest quota with IMF and due to this
status India was allotted a permanent place in Executive Board of Directors.
Which of the statements given above is/are correct?
a) 1 only
b) 2 only
c) Both
d) None
Q.158) Solution (c)
IMF works on quota system, Countries contribute funds to a pool through a quota system from
which countries experiencing balance of payments difficulties can borrow money. IMF is
controlled and managed by a Board of Governors. Currently there are 24 Directors and each
representing a single country or a group of countries. The transactions of IMF are expressed in

www.iasbaba.com

83

Economics- Subject wise compilation of 60 Day Plan- 2016


Special Drawing Right (SDR) which is also known as Paper Gold. The IMF's financial year is form
May 1 to April 30.
India is the founder member of the IMF and also among the top 10 members of IMF, along with
the U.S, Japan, France, Germany, Italy, the United Kingdom, China and Russia. IMF has played an
important role in Indian economy. IMF has provided economic assistance from time to time to
India and has also provided appropriate consultancy in determination of various policies in the
country. India was among the first five nations having the highest quota with IMF and due to
this status India was allotted a permanent place in Executive Board of Directors.
Q.159) Ulaanbaatar Declaration deals with
a) the 11th edition of the Asia-Europe Meeting Summit - "20 Years of ASEM: Partnership for
the Future through Connectivity
b) a prescription on how a duty-centred system, which had been the origin of Indian
philosophy of life, has become the most relevant today.
c) 51 sacred points for betterment of mankind that will start new discourse not only in India
but around the world.
d) Both (b) and (c)
Q.159) Solution (a)
Link:
http://www.newindianexpress.com/world/11th-ASEM-Summit-to-sign-UlaanbaatarDeclaration-today/2016/07/16/article3532245.ece
Q.160) Consider the following statements:
1) UDAY or Ujwal Discom Assurance Yojna was launched to help loss-making discoms turn
around financially, with support from their State governments.
2) Under the scheme, States will take over three-fourths of the debt of their respective
discoms. The governments will then issue UDAY bonds to banks and other financial
institutions to raise money to pay off the banks.
Which of the statements given above is/are correct?
a) 1 only
b) 2 only
c) Both
d) None
Q.160) Solution (c)
UDAY or Ujwal Discom Assurance Yojna was launched in November 2015 to help loss-making
discoms turn around financially, with support from their State governments.
Under the scheme, States will take over three-fourths of the debt of their respective discoms.
The governments will then issue UDAY bonds to banks and other financial institutions to raise
money to pay off the banks.

www.iasbaba.com

84

Economics- Subject wise compilation of 60 Day Plan- 2016


The remaining 25 per cent of the discom debt will be dealt within one of the two ways
conversion into lower interest rate loans by the lending banks or be funded by money raised
through discom bonds backed by State guarantee. Backing from the State will help bring down
the interest rate for the discoms.
In return for the bailout, the discoms have been given target dates (2017 to 2019) by which they
will have to meet efficiency parameters such as reduction in power lost through transmission,
theft and faulty metering, installing smart meters and implementing GIS (geographic
information system) mapping of loss making areas. States will also have to ensure that power
tariffs are revised regularly.
Q.161) Which of the below statement(s) is/are correct about Minimum Alternate Tax (MAT) in
India?
1) The MAT is the means through which the government makes companies that do not pay any
tax cough up some money to the exchequer.
2) Only domestic companies in India, including non-corporate entities, fall under the MAT
provision.
Select the correct code:
a) 1 only
b) 2 only
c) Both
d) None
Q.161) Solution (a)
The MAT is the means through which the government makes companies that do not pay any tax
despite showing hefty book profits and paying handsome dividends to their shareholders
cough up some money to the exchequer. All companies in India, whether domestic or foreign,
fall under this provision. MAT was later extended to cover non-corporate entities as well.
Q.162) Consider the following about Pradhan Mantri Kaushal Vikas Yojana (PMKVY).
1. Pradhan Mantri Kaushal Vikas Yojana (PMKVY) is the flagship outcome-based skill training
scheme of the new Ministry of Skill Development & Entrepreneurship (MSDE).
2. The scheme will be implemented by the Labour Bureau of India.
Select the correct answer about PMKVY:
a) 1 only
b) 2 only
c) Both 1 and 2
d) None of the above
Q.162 Solution (a)

www.iasbaba.com

85

Economics- Subject wise compilation of 60 Day Plan- 2016


Pradhan Mantri Kaushal Vikas Yojanais a unique initiative by the Government of India that aims
to offer 24 lakh Indian youth meaningful, industry relevant, skill based training. Under this
scheme, the trainees will be offered a financial reward and a government certification on
successful completion of training and assessment, which will help them in securing a job for a
better future.
The scheme will be implemented through the National Skill Development Corporation (NSDC).
Further reading:
http://www.skilldevelopment.gov.in/pmkvy.html
http://pmkvyofficial.org/Index.aspx
Q.163) Minimum Support Price (MSP) is a form of market intervention by the Government of
India to insure agricultural producers against any sharp fall in farm prices. Which of the
following is true about MSP?
a) The minimum support prices are announced by the Government of India at the end of the
sowing season for certain crops on the basis of the recommendations of the Commission for
Agricultural Costs and Prices (CACP).
b) In case the market price for the commodity falls below the announced minimum price due
to bumper production and glut in the market, government agencies purchase a minimum
quantity specified at the announced minimum price.
c) Minimum support prices are currently announced for 25 commodities.
d) Government announces two types of administered prices: Minimum Support Prices (MSP)
and Procurement Prices.
Q.163) Solution (c)
MSP is ever a hot topic and a question is a probability always.
Minimum Support Price (MSP) is a form of market intervention by the Government of India to
insure agricultural producers against any sharp fall in farm prices. The minimum support prices
are announced by the Government of India at the beginning of the sowing season for certain
crops on the basis of the recommendations of the Commission for Agricultural Costs and Prices
(CACP).
Q.164) Which of the following is correctly matched?
a) Lorenz Curve Poverty estimation
b) J Curve Taxation
c) Laffer Curve Devalutaion
d) Philips Curve Inflation and unemployment
Q.164) Solution (d)
In economics, the Lorenz curve is a graphical representation of the distribution of income or of
wealth. It was developed by Max O. Lorenz in 1905 for representing inequality of the wealth
distribution.
The J-curve effect is seen in economics when a country's trade balance initially worsens
following a devaluation or depreciation of its currency.

www.iasbaba.com

86

Economics- Subject wise compilation of 60 Day Plan- 2016

The Phillips curve represents the relationship between the rate of inflation and the
unemployment rate.
In economics, the Laffer curve is a representation of the relationship between rates of
taxation and the resulting levels of government revenue.

Q.165) Consider the following about inflation.


1. When the government doesnt attempt to curb price rise allowing the free market
mechanism to function it is said to be open inflation.
2. When international bodies interrupts a price rise in a country through price control
measures and subsidies it is said to be suppressed or controlled inflation.
Select the correct answer:
a) 1 only
b) 2 only
c) Both 1 and 2
d) None of the above
Q.165) Solution (a)
There is no international body acting nationally to the minute idea of controlling inflation by
policy action. They can only advise.
Inflation is often open and suppressed. Inflation is open when markets for goods or factors of
production are allowed to function freely, setting prices of goods and factors without normal
interference by the authorities. Thus open inflation is the result of the uninterrupted operation
of the market mechanism.
There are no checks or controls on the distribution of commodities by the government. Increase
in demand and shortage of supplies persists which tend to lead to open inflation. Unchecked
open inflation ultimately leads to hyper-inflation.
Suppressed Inflation: On the contrary when the government imposes physical and monetary
controls to check open inflation, it is known as repressed or suppressed inflation. The market
mechanism is not allowed to function normally by the use of licensing, price controls and
rationing in order to suppress extensive rise in prices.
Q.166) Consider the following about revenue and capital receipts.
1. Interest receipts, dividends and profits are parts of revenue receipts.
2. Article 112 of the constitution mandates expenditure is shown in revenue and other
categories.
3. Disinvestment proceeds are part of capital receipts.
Select the correct answer
a) 1 and 2 only
b) 1, 2 and 3 only
c) 1 and 3 only

www.iasbaba.com

87

Economics- Subject wise compilation of 60 Day Plan- 2016


d) All of the above
Q.166) Solution (b)
Further reading: http://pib.nic.in/budget2015/book/Budgetary.pdf
Q.167) The Government of India has approved a policy framework for private participation in
rail connectivity and capacity augmentation projects, put forward by the Ministry of Railways.
The policy contains which of the following models?
1. Non-Government Railway
2. Build/Operate/Transfer (BOT)
3. Maintenance Management Contract
4. Joint Venture with equity participation by Railways
5. Design-Build-Operate (DBO)
Select the correct answer using the codes given below:
a) 2, 3 and 4 only
b) 1, 2 and 4 only
c) 2, 3, 4 and 5 only
d) 1, 2, 3, 4 and 5
Q.167) Solution (d)
Ministry is now making joint ventures with State governments for an equity participation. So the
same question 3 years back would have been b but today is d.
Q.168) Consider the following about Agroforestry.
1. It is a dynamic, ecologically based, natural resource management system that, diversifies
and sustains production and builds social institutions.
2. Its objective is to Encourage and expand tree plantation in complementarity and integrated
manner with crops and livestock to improve productivity, employment, income and
livelihoods of rural households, especially the small holder farmers.
3. Agroforestry has a lot in common with intercropping. Both have two or more plant species
(such as nitrogen-fixing plants) in close interaction, both provide multiple outputs.
4. National Agroforestry Policy is under the nodal ministry of Environment Forests and Climate
Change.
Which of the following are correct about Agroforestry
a) 2, 3 and 4 only
b) 1 and 4 only
c) 1, 2 and 3 only
d) All of the above
Q.168) Solution (c)
With the budget of 2016 giving emphasis on agriculture, a conceptual question is expected.

www.iasbaba.com

88

Economics- Subject wise compilation of 60 Day Plan- 2016


Agroforestry is defined as a land use system which integrates trees and shrubs on farmlands and
rural landscapes to enhance productivity, profitability, diversity and ecosystem sustainability. It
is a dynamic, ecologically based, natural resource management system that, through integration
of woody perennials on farms and in the agricultural landscape, diversifies and sustains
production and builds social institutions.
Major policy initiatives, including the National Forest Policy 1988, the National Agriculture Policy
2000, Planning Commission Task Force on Greening India 2001, National Bamboo Mission 2002,
National Policy on Farmers, 2007 and Green India Mission 2010, emphasize the role of
agroforestry for efficient nutrient cycling, organic matter addition for sustainable agriculture
and for improving vegetation cover. However, agroforestry has not gained the desired
importance as a resource development tool due to various factors.
Q.169) Consider the following:
1. Department of Industrial Policy and Promotion (DIPP) has permitted investments from
Pakistan in sectors/activities other than defence, space and atomic energy.
2. Recently Pakistan has moved from a Positive List regime to a Negative List regime and now
subsequently awarded MFN status to India.
Select the correct answer from the codes given below:
a) Only 1
b) Only 2
c) Both 1 and 2
d) Neither 1 nor 2
Q.169) Solution (a)
We are still in negotiation for Non discriminatory market access as the term MFN has issues in
Pakistan.
Q.170) To check the menace of counterfeiting of banknotes, the RBI has initiated several
measures. Which of the following are parts of these measures?
1. New security features were added to banknotes in all denominations during the year 200506.
2. Instructions have been issued to banks to disburse only sorted and genuine notes through
their counters/ATMs
3. RBI regularly conducts training programmes on detection of counterfeit notes for
employees/officers of banks and other organizations handling large amount of cash.
4. Posters on Know Your Banknotes are also displayed at the bank branches.
Select the correct answer from the codes given below:
a) 1 and 2
b) 2 and 4
c) 1, 2 and 3
d) 1, 2, 3 and 4

www.iasbaba.com

89

Economics- Subject wise compilation of 60 Day Plan- 2016

Q.170) Solution (d)


Self explanatory
Q.171) With reference to the concept of Food Security as enunciated in the FAO World Food
Summit-1996, which of the following are the integral components of Food Security?
1. The availability of sufficient quantities of food of appropriate quality, supplied through
domestic production or imports, including food aid.
2. Access by individuals to adequate resources for acquiring appropriate foods for a nutritious
diet.
3. Utilization of food through adequate diet, clean water, sanitation and health care to reach a
state of nutritional well-being where all physiological needs are met.
4. The freedom to choose personally and culturally acceptable foods
Select the correct answer from the codes given below:
a) 1 and 2
b) 1 and 3
c) 1, 2 and 3
d) All the above
Q.171) Solution (c)
Food availability: The availability of sufficient quantities of food of appropriate quality, supplied
through domestic production or imports (including food aid).
Food access: Access by individuals to adequate resources (entitlements) for acquiring
appropriate foods for a nutritious diet. Entitlements are defined as the set of all commodity
bundles over which a person can establish command given the legal, political, economic and
social arrangements of the community in which they live (including traditional rights such as
access to common resources).
Utilization: Utilization of food through adequate diet, clean water, sanitation and health care to
reach a state of nutritional well-being where all physiological needs are met. This brings out the
importance of non-food inputs in food security.
Stability: To be food secure, a population, household or individual must have access to adequate
food at all times. They should not risk losing access to food as a consequence of sudden shocks
(e.g. an economic or climatic crisis) or cyclical events (e.g. seasonal food insecurity). The concept
of stability can therefore refer to both the availability and access dimensions of food security
Q.172) Consider the following statements regarding Regional rural Banks (RRBs):
1. They provide direct loans to small and marginal farmers.
2. They are co-sponsored by the Reserve Bank of India.
3. They also perform other banking operations.
4. State Governments are share holders in RRBs.
Which of the above statements are correct about RRBs?

www.iasbaba.com

90

Economics- Subject wise compilation of 60 Day Plan- 2016


a)
b)
c)
d)

1,2 and 3
2,3 and 4
1,3 and 4
All of the above

Q.172) Solution (c)


Regional Rural Banks (also RRBs) are local level banking organizations operating in different
States of India. They have been created with a view to serve primarily the rural areas of India
with basic banking and financial services. However, RRBs may have branches set up for urban
operations and their area of operation may include urban areas too.
The area of operation of RRBs is limited to the area as notified by Government of India covering
one or more districts in the State. RRBs also perform a variety of different functions.
RRBs perform various functions in following heads
Providing banking facilities to rural and semi-urban areas. Carrying out government
operations like disbursement of wages of MGNREGA workers, distribution of pensions etc.
Providing Para-Banking facilities like locker facilities, debit and credit cards.
The Regional Rural Bank were owned by the Central Government, the State Government and
the Sponsor Bank(There were five commercial banks, Punjab National Bank, State Bank of India,
Syndicate Bank, United Bank of India and United Commercial Bank, which sponsored the
regional rural banks) who held shares in the ratios as follows Central Government-50%, State
Government- 15% and Sponsor Banks- 35%. Earlier, Reserve Bank of India had laid down ceilings
on the rate of interest to be charged by these RRBs.
Q.173) Ultra Mega Power projects are a Government of India initiatives. Consider the
following statements regarding UMPP?
1. Every Ultra Mega Power Plant will have a capacity to generate more than 10,000 MW.
2. They are all hydro power projects.
Which of the above statements are correct?
a) 1 only
b) 2 only
c) Both 1 and 2
d) Neither 1 nor 2
Q.173) Solution (d)
Ultra Mega Power Projects (UMPP) are a series of ambitious power stations planned by the
Government of India. This would entail the creation of an additional capacity of at least 100,000
MW by 2022. Ultra Mega Power projects, each with a capacity of 4000 MW or above, are being
developed with the aim of bridging this gap. These projects will be coal based thermal power
plants.
Q.174) Disguised Unemployment refers to:
a) Persons with no jobs

www.iasbaba.com

91

Economics- Subject wise compilation of 60 Day Plan- 2016


b) More people employed for a job which a few can accomplish.
c) Unemployment amongst women
d) Unemployment of people above 60 years of age
Q.174) Solution (b)
Disguised unemployment is a kind of unemployment in which some people look like being
employed but are actually not employed fully.
Disguised unemployment exists where part of the labor force is either left without work or is
working in a redundant manner where worker productivity is essentially zero. It is
unemployment that does not affect aggregate output.
Q.175) India has .. % of worlds geographical area and % of the worlds population
(according to 2011 census).
a) 4.8, 16
b) 2.1, 18
c) 2.4, 17
d) 1.8, 15
Q.175) Solution (c)
Self Explanatory
Q.176) Consider the following statements:
1. The theory that says that the growth will percolate down to all sectors is called
Decentralised Growth theory.
2. The growth will percolate by creation of jobs and increase in demand for labour.
Which of the above statements are correct?
a) 1 only
b) 2 only
c) Both 1 and 2
d) Neither 1 nor 2
Q.176) Solution (b)
The theory is called Trickle down theory. It is based on a centralized model and not a
decentralized one.
Q.177) Which of the below given statements is/are true about Value Added Tax (VAT)?
1) VAT is a type of consumption tax.
2) The amount of value-added tax that the user pays is the cost of the product, less any of the
costs of materials used in the product that have already been taxed.
Select the correct code:
a) 1 only

www.iasbaba.com

92

Economics- Subject wise compilation of 60 Day Plan- 2016


b) 2 only
c) Both
d) None
Q.177) Solution (c)
Link: http://www.investopedia.com/terms/v/valueaddedtax.asp
Q.178) Which among the following definition(s) is/are incorrectly matched?
a) Revenue Deficit : : difference between the revenue receipts and the revenue expenditure
b) Budget Deficit : : when total expenditure exceeds total receipts, where, total expenditure
includes aggregate of both revenue expenditure and capital expenditure. Like wise, total
receipts includes both revenue receipts and capital receipts.
c) Fiscal Deficit : : determined by arriving at the gap between the governments total income
and expenditure after excluding interest savings as well as interest payments
d) None of the above
Q.178) Solution (c)
Fiscal Deficit: The term fiscal deficit may be defined as budgetary deficit plus market borrowings
and other liabilities of the govt. of India. In other words, fiscal deficit equals revenue receipts
plus non-debt capital receipts mines total expenditure.
Primary Deficit: Primary deficit is determined by arriving at the gap between the governments
total income and expenditure after excluding interest savings as well as interest payments.

Q.179) Which of the following are correctly matched?


1)
2)
3)
4)

Government borrowings from the Reserve Bank and treasury bills- Capital Account
Different proceeds of taxes levied by the government - Revenue Account
Market loans raised by the government - Capital Account
Divestment of equity holding in public sector enterprises - Revenue Account

Select the correct code:


a)
b)
c)
d)

1 and 2 only
1, 2 and 4 only
1, 2 and 3 only
All of the above

*Note: Question is corrected, please update the same.


Q.179) Solution (c)
Divestment of equity holding in public sector enterprises - Capital Account (not Revenue
Account)

www.iasbaba.com

93

Economics- Subject wise compilation of 60 Day Plan- 2016

Q.180) What is the difference between quaternary and quinary activities?


a) While, quaternary activities are related to IT enabled services, quinary activities are related
to financial services.
b) Quaternary activities pertain to information processing. Whereas, quinary activities pertain
to decision making.
c) Both a and b
d) None of the above
Q.180) Solution (b)
Tertiary activities are categorized into quaternary and quinary activities. The former involves
collection, production, processing, and dissemination of information. For example, software
developers, tax consultants etc.
Quinary activities focus on creation, re-arrangement, and interpretation of ideas. It pertains to
high level decision making. Therefore, it is also called gold collar professions. For example,
business executives, research scientists etc.
Q.181) Which of the following factors determine the location of an industry?
1) Market
2) Population density
3) Industrial policy
4) Climate of a region
Select the correct code:
a) 1 and 2 only
b) 2 and 3 only
c) 1, 3 and 4 only
d) All of the above
Q.181) Solution (d)
The most important factors are:
(i)
Raw material.
(ii) Source of Power.
(iii) Labour.
(iv) Means of transportation.
(v) Market.
(vi) Other factors like climate, Government Policies, capital, water, land etc.
Markets provide the outlets for manufactured products. Heavy machine, machine tools, heavy
chemicals are located near the high demand areas as these are market orientated.

www.iasbaba.com

94

Economics- Subject wise compilation of 60 Day Plan- 2016


Certain agriculture raw materials are limited by the climate, e.g. equatorial climate rubber for
processing.
Population density decides the kind of labour and market is available.
Q.182) Identify the state that witnessed the lowest services growth in India
b) Jharkhand
c) Odisha
d) Bihar
e) Jammu & Kashmir
Q.182) Solution (d)
Jammu and Kashmir had the lowest services growth at 2.0 per cent low and negative
growth in most of the sectors except public administration.
Bihars services sector growth was among the fastest with a consistent double-digit growth
in the last seven years due to high growth in the high weighted sectors like trade, hotels and
restaurants, and real estate and business services besides transport by other means

Q.183) Consider the following w.r.t. Securities and Exchange Board of India (SEBI).
1. SEBI was established in 1988 and was given statutory powers in 1992 under the
Securities and Exchange Board of India act 1992.
2. The Securities Laws (Amendment) Act, 2014 provides for search and seizure after
obtaining approval from Union Minister of Finance.
3. National Strategy of Financial Education was initiated by SEBI with a vision of a
financially and empowered India.
4. SEBI is part of Financial Stability and Development Council (FSDC).
Select the correct answer
a) 1, 2 and 4 only
b) 1 and 3 only
c) 1, 3 and 4 only
d) 2, 3 and 4 only
*Note: Statement 1 in this question is corrected, please update the same
Q.183) Solution (c)
It isnt MoF but a magistrate or Judge of a designated court in Mumbai. Rest of the
statements are correct. Yes SEBI also has a role in financial inclusion.
Refer Chapter Finance, India Year Book 2016.

Q.184) Which of the following is incorrect about FIPB?


www.iasbaba.com

95

Economics- Subject wise compilation of 60 Day Plan- 2016

a) The Foreign Investment Promotion Board (FIPB), presently is housed in the


Department of Economic Affairs, Ministry of Finance, is an inter-ministerial body.
b) The FIPB was initially constituted under the Prime Minister's Office (PMO) in the
wake of the economic liberalization drive of the early 1990s.
c) The Board was reconstituted in 1996 with transfer of the FIPB to DIPP
d) The approval limit for FIPB is Rs. 2000crore beyond which it needs approval of Union
Cabinet.
Q.184) Solution (d)
http://fipb.gov.in/AboutUs.aspx
The information here with approval limits is a little outdated. The government raised it to
3000 crores and beyond which it is approved by CCEA and not cabinet.
**As per the Consolidated FDI Policy, June 2016, the approval limit for FIPB has been further
increased from 3000 crores to Rs. 5000 crores.
Link: http://www.thehindu.com/business/Industry/fipb-clears-fdi-proposals-worth-rs13000crore/article8537839.ece
Q.185) Consider the following statements regarding Treasury bills:
1. Treasury bills are issued by the RBI on behalf of the government of India.
2. Treasury bills are issued through auctions and for a period of 91 days only.
3. State governments are allowed to issue Treasury bills.
4. Treasury bills are available for a minimum amount of 1 lakh and then in multiples of
25000.
Which of the above statements are incorrect?
a) 1 only
b) 2,3 and 4
c) 1,2 and 3
d) None of the above

Q.185) Solution (b)


Treasury Bill Market (T - Bills): This market deals in Treasury Bills of short term duration issued by RBI on behalf of
Government of India. At present three types of treasury bills are issued through
auctions, namely 91 day, 182 day and364day treasury bills. State government does not
issue any treasury bills.
Interest is determined by market forces. Treasury bills are available for a minimum
amount of Rs. 25,000 and in multiples of Rs. 25,000. Periodic auctions are held for their
Issue.

www.iasbaba.com

96

Economics- Subject wise compilation of 60 Day Plan- 2016

T-bills are highly liquid, readily available; there is absence of risk of default. In India Tbills have narrow market and are undeveloped. Commercial Banks, Primary Dealers,
Mutual Funds, Corporates, Financial Institutions, Provident or Pension Funds and
Insurance Companies can participate in T-bills market.

Doubts: In explanation, you have provided State government does not issue any treasury
bills. But in previous test you said state government issues 14 day treasury bills. Please ratify
babaji.
Clarifications:
With treasury bills... The idea that state government doesn't issue is correct... But 14 day
bills are issued by the RBI to state governments and nobody else... So it is not by state
government but to state government... Pls make the change...

Q.186) Most of the trading in the Indian stock market takes place on its two stock
exchanges: the Bombay Stock Exchange (BSE) and the National Stock Exchange (NSE). The
BSE has been in existence since 1875. The NSE, on the other hand, was founded in 1992
and started trading in 1994. Which of the following statements are correct about the stock
exchanges of India?
1. Both exchanges follow the same trading mechanism, trading hours and settlement
process.
2. The number of firms listed in NSE is almost three times than they are listed in BSE.
3. NSE enjoys almost a complete monopoly in Derivative trading.
Select the code from following:
a) 1 and 2
b) 2 and 3
c) 1 and 3
d) All of the above
Q.186) Solution (c)
The number of firms listed in NSE is almost one third than that of BSE.
Doubts:
Q19 Statement 1. This is wrong.
Both BSE and NSE follow different trading mechanism.
For a common man it is online trading.
But an upsc aspirant is not expected to be a common man/woman!! :)
Please see the following. BSE follows BOLT ie bombay online trading system... NSE Follows
NEAT. Also since NSE is exclusively dealing with future and options, this also adds into
difference between the two trading mechanism.
http://articles.economictimes.indiatimes.com/2009-03-25/news/28385765_1_tradingplatform-nse-bse-platform

www.iasbaba.com

97

Economics- Subject wise compilation of 60 Day Plan- 2016

Here it means to say that they have used new techniques for integration but principally the
system of two is different.
Clarifications:
We appreciate your efforts Kunal Krishna, but go through other recent sources as well.
IASbaba sticks to the same answer Both exchanges follow the same trading mechanism,
trading hours and settlement process
Refer: http://www.investopedia.com/articles/stocks/09/indian-stock-market.asp
Or
http://stockmarketsignals.com/complete-overview-indian-stock-market/

www.iasbaba.com

98

S-ar putea să vă placă și